Download as pdf or txt
Download as pdf or txt
You are on page 1of 155

STUDENT SUPPOR

SESSION : 2020-21

MATHEMATICS

KENDRIYA VIDYALAYA SANGATHAN, NEW DELHI


STUDENT SUPPORT MATERIAL
(Revised)

Class –X

Mathematics

Session: 2020-21

KENDRIYA VIDYALAYA SANGATHAN


NEW DELHI

2|Page
A WORD TO MY DEAR STUDENTS

It gives me great pleasure in presenting the Students „Support Material to all KV students of
class X. The material has been prepared keeping in mind your needs when you are preparing
for final exams and wish to revise and practice questions or when you want to test your ability
to complete the question paper in the time allotted or when you come across a question while
studying that needs an immediate answer but going through the textbook will take time or
when you want to revise the complete concept or idea in just a minute or try your hand at a
question from a previous CBSE Board exam paper or the Competitive exam to check your
understanding of the chapter or unit you have just finished. This material will support you in
any way you want to use it.
A team of dedicated and experienced teachers with expertise in their subjects has prepared
this material after a lot of exercise. Care has been taken to include only those items that are
relevant and are in addition to or in support of the textbook. This material should not be taken
as a substitute to the NCERT textbook but it is designed to supplement it.
The Students „Support Material has all the important aspects required by you; a design of the
question paper, syllabus, all the units/chapters or concepts in points, mind maps and
information in tables for easy reference, sample test items from every chapter and question
papers for practice along with previous years Board exam question papers.
I am sure that the Support Material will be used by both students and teachers and I am
confident that the material will help you perform well in your exams.
Happy learning!
FOREWORD
The Students' Support Material is a product of an in-house academic exercise undertaken by our
subject teachers under the supervision of subject expert at different levels to provide the students
a comprehensive, yet concise, learning support tool for consolidation of your studies. It
consists of lessons in capsule form, mind maps, concepts with flowcharts, pictorial representation
of chapters wherever possible, crossword puzzles, question bank of short and long answer type
questions with previous years' CBSE question papers.

The material has been developed keeping in mind latest CBSE curriculum and question
paper design. This material provides the students a valuable window on precise information and it
covers all essential components that are required for effective revision of the subject.
In order to ensure uniformity in terms of content, design, standard and presentation of the
material, it has been fine-tuned at KVS HQRS level.

I hope this material will prove to be a good tool for quick revision and will serve the purpose
of enhancing students „confidence level to help them perform better. Planned study blended with
hard work, good time management and sincerity will help the students reach the pinnacle of
success.

Best of Luck.
Mathematics

STUDENT SUPPORT MATERIAL


ADVISORS

Shri U.N Khaware,

KVS (HQ), New Delhi. KVS (HQ), New Delhi.

CO-ORDINATION TEAM KVS (HQ)

Smt. Indu Kaushik, Deputy Commissioner(Acad),KVS (HQ),New Delhi.

CONTENT TEAM

Shri O.P.Sharma, PGT (Mathematics), KV Patna Region..

Shri S.K.Singh, TGT(Maths), KV Patna Region.

REVIEW TEAM

Shri Sunil Kumar (Principal),KV Sitapur.

Shri Kamal Kumar, TGT(Maths),KV Sitapur.


Smt AbhaKumari, TGT(Maths),KV Sitapur.
INDEX
S.NO TOPIC
PART -1
1 Real Numbers
2 Polynomials
3 A pair of linear equations in two variables
4 Quadratic Equations
5 Arithmetic Progressions
6 Triangles
7 Coordinate Geometry
8 Introduction to Trigonometry
9 Some Applications of Trigonometry
10 Circles
11 Constructions
12 Area Related to Circles
13 Surface Areas and Volumes
14 Statistics
15 Probability
PART – 2
16 Activities
PART- 3
17 Model Question Paper 1 (Standard) with marking scheme
18 Model Question Paper 2 (Basic) with blue print and marking scheme
COURSE STRUCTURE CLASS –X

Units Unit Name Marks


I NUMBER SYSTEMS 06
II ALGEBRA 20
III COORDINATE GEOMETRY 06
IV GEOMETRY 15
V TRIGONOMETRY 12
VI MENSURATION 10
VII STATISTICS & PROBABILTY 11
Total 80

UNIT I: NUMBER SYSTEMS

1. REAL NUMBER (8) Periods


Fundamental Theorem of Arithmetic - statements after reviewing work done earlier
and after illustrating and motivating through examples, Proofs of irrationality of
Decimal representation of rational numbers interms of terminating/non-
terminating recurring decimals.

UNIT II: ALGEBRA

1. POLYNOMIALS (4) Periods

Zeros of a polynomial. Relationship between zeros and coefficients of quadratic


polynomials.

2. PAIR OF LINEAR EQUATIONS IN TWO VARIABLES (11) Periods

Pair of linear equations in two variables and graphical method of their


solution, consistency/inconsistency.
Algebraic conditions for number of solutions. Solution of a pair of linear equations in
two variables algebraically - by substitution, by elimination. Simple situational
problems. Simple problems on equations reducible to linear equations.

3. QUADRATIC EQUATIONS (10) Periods


2
Standard form of a quadratic equation ax + bx + c = 0, (a ≠ 0). Solutions of quadratic
equations (only real roots) by factorization, and by using quadratic formula.
Relationship between discriminant and nature of roots.

4. ARITHMETIC PROGRESSIONS (4) Periods


th
Motivation for studying Arithmetic Progression Derivation of the n term and sum of
the first n terms of A.P.

5
UNIT III: COORDINATE GEOMETRY

1. LINES (In two-dimensions) (10) Periods

Review: Concepts of coordinate geometry, graphs of linear equations. Distance


formula.
Section formula (internal division).

UNIT IV: GEOMETRY

1. TRIANGLES (10) Periods

Definitions, examples, counter examples of similar triangles.


1. (Prove) If a line is drawn parallel to one side of a triangle to intersect the other two
sides in distinct points, the other two sides are divided in the same ratio.
2. (Motivate) If a line divides two sides of a triangle in the same ratio, the line is parallel
to the third side.
3. (Motivate) If in two triangles, the corresponding angles are equal, their corresponding
sides are proportional and the triangles are similar.
4. (Motivate) If the corresponding sides of two triangles are proportional, their
corresponding angles are equal and the two triangles are similar.
5. (Motivate) If one angle of a triangle is equal to one angle of another triangle and the
sides including these angles are proportional, the two triangles are similar.
6. (Motivate) If a perpendicular is drawn from the vertex of the right angle of a right
triangle to the hypotenuse, the triangles on each side of the perpendicular are similar
to the whole triangle and to each other.
7. (Prove) In a right triangle, the square on the hypotenuse is equal to the sum of the
squares on the other two sides.

2. CIRCLES (8) Periods


Tangent to a circle at, point of contact

1. (Prove) The tangent at any point of a circle is perpendicular to the radius through the
point of contact.

2. (Prove) The lengths of tangents drawn from an external point to a circle are equal.

6
3. CONSTRUCTIONS (4) Periods

1. Division of a line segment in a given ratio (internally).


2. Tangents to a circle from a point outside it.

UNIT V: TRIGONOMETRY
1. INTRODUCTION TO TRIGONOMETRY (8) Periods

Trigonometric ratios of an acute angle of a right-angled triangle. Proof of their


0 0 0
existence (well defined). Values of the trigonometric ratios of 30 , 45 and 60 .
Relationships between the ratios.

2. TRIGONOMETRIC IDENTITIES (10) Periods


2 2
Proof and applications of the identity sin A + cos A = 1. Only simple identities to be
given.

3. HEIGHTS AND DISTANCES: Angle of elevation, Angle of Depression. (8) Periods

Simple problems on heights and distances. Problems should not involve more
than two right triangles. Angles of elevation / depression should be only 30°,
45°, 60°.

UNIT VI: MENSURATION


1. AREAS RELATED TO CIRCLES (10) Periods
Motivate the area of a circle; area of sectors and segments of a circle. Problems
based on areas and perimeter / circumference of the above said plane figures. (In
calculating area of segment of a circle, problems should be restricted to central angle
of 60°and 90° only. Plane figures involving triangles, simple quadrilaterals and circle
should be taken.)

2. SURFACE AREAS AND VOLUMES (8) Periods

1. Surface areas and volumes of combinations of any two of the following: cubes,
cuboids, spheres, hemispheres and right circular cylinders/cones.

2. Problems involving converting one type of metallic solid into another and other
mixed problems. (Problems with combination of not more than two different
solids be taken).

UNIT VII: STATISTICS AND PROBABILITY

1. STATISTICS (10) Periods


Mean, median and mode of grouped data (bimodal situation and step deviation method
for finding the mean to be avoided).

2. PROBABILITY (10) Periods


Classical definition of probability. Simple problems on finding the probability of an
event.
CHAPTER TOPICS REMOVED
UNIT I – NUMBER SYSTEMS
REAL NUMBERS : Euclid’s division lemma
UNIT II - ALGEBRA
POLYNOMIALS :
Statement and simple problems on division algorithm for polynomials with real coefficients.
PAIR OF LINEAR EQUATIONS IN TWO VARIABLES: cross multiplication method
QUADRATIC EQUATIONS :
Situational problems based on equations reducible to quadratic equations
ARITHMETIC PROGRESSIONS :
Application in solving daily life problems based on sum to n terms
UNIT III – COORDINATE GEOMETRY
COORDINATE GEOMETRY :
Area of a triangle.
UNIT IV - GEOMETRY
TRIANGLES : Proof of the following theorems are deleted
The ratio of the areas of two similar triangles is equal to the ratio of the
squares of their corresponding sides.
In a triangle, if the square on one side is equal to sum of the squares on the
other two sides, the angle opposite to the first side is a right angle.
CIRCLES No deletion
CONSTRUCTIONS Construction of a triangle similar to a given triangle.
UNIT V - TRIGONOMETRY
INTRODUCTION TO TRIGONOMETRY :motivate the ratios whichever are defined at 0o and 90o
TRIGONOMETRIC IDENTITIES Trigonometric ratios of complementary angles.
HEIGHTS AND DISTANCES No deletion
UNIT VI - MENSURATION
AREAS RELATED TO CIRCLES : Problems on central angle of 120°
SURFACE AREAS AND VOLUMES Frustum of a cone.
UNIT VI - STATISTICS & PROBABILITY
STATISTICS Step deviation Method for finding the mean
Cumulative Frequency graph
PROBABILITY No deletion
MATHEMATICS-Standard
QUESTION PAPER DESIGN
CLASS – X (2020-21)
Time : 3 Hours Max. Marks: 80

%
S. Total
Typology of Questions Weightage
No. Marks
(approx.)

Remembering: Exhibit memory of previously learned material by


recalling facts, terms, basic concepts, and answers.
43 54
1
Understanding: Demonstrate understanding of facts and ideas by
organizing, comparing, translating, interpreting, giving descriptions, and
stating main ideas

Applying: Solve problems to new situations by applying acquired 19 24


2
knowledge, facts, techniques and rules in a different way.

Analysing :
Examine and break information into parts by identifying motives or
causes. Make inferences and find evidence to support generalizations

Evaluating:
Present and defend opinions by making judgments about information, 18 22
3
validity of ideas, or quality of work based on a set of criteria.

Creating:
Compile information together in a different way by combining elements
in a new pattern or proposing alternative solutions

80 100
Total

INTERNAL ASSESSMENT 20 MARKS


Pen Paper Test and Multiple Assessment (5+5) 10 Marks
Portfolio 05 Marks
Lab Practical (Lab activities to be done from the prescribed books) 05 Marks
MATHEMATICS-Basic
QUESTION PAPER DESIGN
CLASS – X (2020-21)
Time : 3Hours Max. Marks: 80

%
S. Total
Typology of Questions Weightage
No. Marks
(approx.)

Remembering: Exhibit memory of previously learned material by


recalling facts, terms, basic concepts, and answers.
1 60 75
Understanding: Demonstrate understanding of facts and ideas by
organizing, comparing, translating, interpreting, giving descriptions, and
stating main ideas

Applying: Solve problems to new situations by applying acquired 12 15


2
knowledge, facts, techniques and rules in a different way.

Analysing :
Examine and break information into parts by identifying motives or
causes. Make inferences and find evidence to support generalizations

Evaluating:
Present and defend opinions by making judgments about information, 8 10
3
validity of ideas, or quality of work based on a set of criteria.

Creating:
Compile information together in a different way by combining elements
in a new pattern or proposing alternative solutions

80 100
Total

INTERNAL ASSESSMENT 20 MARKS


Pen Paper Test and Multiple Assessment (5+5) 10 Marks
Portfolio 05 Marks
Lab Practical (Lab activities to be done from the prescribed books) 05 Marks

PRESCRIBED BOOKS:

1. Mathematics - Textbook for class X - NCERT Publication


2. Guidelines for Mathematics Laboratory in Schools, class X - CBSE Publication
3. Laboratory Manual - Mathematics, secondary stage - NCERT Publication
4. Mathematics exemplar problems for class X, NCERT publication.
Real Numbers
(Key Points
Real Numbers

IRRATIONAL

NATURAL WHOLE NUMBERS INTEGER (Z)


NUMBERS(COUNTING
NO 1,2,3,...
(W) (0,1,2,3,4,…)

NEGATIVE
ZERO (0) POSITIVE INTEGERS
(1, 2, 3…)

REPEATING DECIMAL
( 2/5,¾,….) (RATIONAL (RECURRING
NUMBERS) DECIMAL) (1/3,
2/7,3/11,…)
(RATIONAL NUMBERS)

1. The Fundamental theorem of Arithmetic:-


Every composite number can be expressed (factorized) as a product of primes and this factorization is
unique, apart from the order in which the prime factors occur.
Ex.: 24= 2 x 2 x 2 x 3 = 3 x 2 x 2 x 2

Theorem: Let x be a rational number whose decimal expansion terminates. Then x can be expressed in the
n m
form of p/q where are co-prime and the prime factorization of q is of the form of 2 x5 ,
where n,m are non-negative integers.
Ex:-

p
Theorem: x = Let x be a rational number such that the prime factorization of q is not of the form of
q
2nx 5m, where n,m are non-negative integers. Then x has a decimal expansion which is non-
terminating repeating (recurring).
Ex:-

Theorem: For any two positive integers a and b,


HCF (a,b) XLCM (a,b) =aXb

Ex.:4&6; HCF (4, 6) =2, LCM (4,6) =12; HCFXLCM=2X12=24


Ans.: a x b=24

LEVEL-I

1. Write a rational number between and 3


43
2. The decimal expansion of the rational will terminate after how many places
no. 2453

of decimal?
3. Find the (HCF x LCM) for the numbers 100 and 190 by prime factorization method.
p
4. If is a rational number (q ≠ 0). What is the condition on q so that the decimal representation of
q
p
is terminating?
q
5. State whether the number (  )(  ) is rational or irrational. justify.

6. Write one rational and one irrational number lying between 0.25 and 0.32.
7. Express 256 as a product of primes.
51
8. Write whether the rational number will have a terminating decimal expansion or a non
1500
terminating repeating decimal expansion.
9. If m and n are co-prime number , then find the HCF (m,n).

10. Express 0.2545454 .............. as a fraction in simplest form.

LEVEL-II

1. Use the prime factorization method to find the HCF of 1288 and 575.
2. Check whether 5 x 3 x 11+11 and 5 x 7+7 x 3 are composite number and justify.
3. Check whether 6n can end with the digit 0, where n is any natural number.
4. Given that LCM (26,169) =338, write HCF (26,169).
5. Find the HCF and LCM of 6 ,72 and 120 using the prime factorization method.

LEVEL-III
1. Show that √3 is an irrational number.

2. Show that 5+3 is an irrational number

3. Show that is an irrational number, if √3 is an irrational number.

4. Find the LCM & HCF of 26 and 91 and verify that HCF x LCM=product of the two numbers

5. Prove that is irrational

(PROBLEMS FOR SELF EVALUATION/HOTS)


1. State the fundamental theorem of Arithmetic.

2. Express 2658 as a product of its prime factors.

3. Find the LCM and HCF of 17, 23 and 29.

4. Prove that is not a rational number.


5. Find the largest positive integer that will divide 122,150 and 115 leaving remainder 5,7 and
11 respectively.

6. Show that there is no positive integer n for which  is rational.


7. Using prime factorization method, find the HCF and LCM of 72,126 and 168. Also show
that HCF X LCM ≠ product of three numbers.
8. Three sets of English, Mathematics and Science books containing 336, 240 and 96 books
respectively have to be stacked in such away that all the books are stored subject wise and the height
of each stack is the same. How many stacks will be there?
9. A person wanted to distribute 96 apples and 112 oranges among poor children in an orphanage. He
packed all the fruits in boxes in such a way that each box contains fruits of the same variety, and also
every box contains an equal number of fruits.
• Find the maximum number of boxes in which all the fruits can be packed.
• Which concept have you used to find it?
10. Teacher draws the factor tree, given in figure and ask the students to find the value

of x without finding the value of y and z. Shaurya gives the answer x=136

a) Is his answer correct?

b) Give reason for your answer.

17

Answer Level-I
1. 1.42 (flexible)
2. After 4 places of decimal.
3. 19000

4. q is of the form 2n.5m, where m and n are non-negative integers.


5. Rational number
6. One rational number=26/100, one irrational no. =0.27010010001………
7. 28
8. Terminating 9. 1
10.14/55
Level-II

1. 23
2. Composite number and justification show that numbers are having more than two factors
3. No,6n cannot end with the digit 0.
4. 13
5. HCF=6, LCM = 360
LEVEL-III

1. See textbook
2. See textbook ex 1.3 Q.No.-2
3. See textbook
4. LCM = 182 HCF=13 , Product =2366
5. See textbook

Problems for self-evaluation/Hots

1. Write statement of fundamental theorem

2. 2658= 2x3x443

3. HCF= 1 , LCM =11339

4. Refer Ncert book

5. Number =13
7. HCF = 6 LCM = 504 NOT EQUAL

8. Number of books in a stack = 48 (i) No.of boxes=14


9.16 fruits in a box Number of Boxes

=13 Number System &HCF


10. (a) Yes, his answer is correct.
(b) Z =2 X 17 = 34, Y = 2 X 34 = 68, X = 2 x 68 = 136
(c) Knowledge of prime factorization.
3. Write the condition when parabola is open upwards?
4. Write the number of zeroes of the quadratic polynomial.

5. Find the quadratic polynomial whose zeros are √2 +3 and √2 - 3.


6. Write the degree of the zero polynomial.
7. Find the zeros of the quadratic polynomial 6x2-7x-3 and verify the relationship between
the zeros and the coefficients.
8. Find the quadratic polynomial sum of whose zeros is 2√3 and their product is 2.

Level II
9. If the sum of squares of zeros of the polynomial 6x2+x+k is 25/36 find the value of k.
10. If one zero of the quadratic polynomial f(x) =4x2-8kx-9 is negative of the other, find the
value of k.
11. Find the value of k for which the quadratic polynomial 9x2-3kx+k has equal zeros
12. If the sum and product of zeros of the polynomial ax2-5x+c is equal to 10 each,
find the values of a and c.

13. Find all the zeros of polynomial x4+x3-9x2-3x+18 if the two of its zeros are √3 and -√3.
14.If α and β are the zeros of quadratic polynomial p(x) =x2-(k-6) x+ (2k+1)
Find the value of k if α+þ=αβ.
15. If the zeros of polynomial x2-5x+k are the reciprocal of the zeros then find the value of k.
16. If α and β are the zeros of the quadratic polynomial x2 -6x +a find the value of „a‟
If 3α +2β=20

LEVEL III
17. If α and β are the zeros of the quadratic polynomial 3x2 -10x +7, find the value of
α2 +β2
18. If α and β are the zeroes of a polynomial x2-x-30, then form a quadratic
Polynomial whose zeroes are 2-α and 2- β

20
26. If the number of apples and mangoes are the zeroes of polynomial 3x 2-8x-2k+1.
and the number of apples are 7 times the number of mangoes,
then find the number of zeroes and value of k.
ANSWERS

LEVEL I
1. A) 1 polynomial x-1
B) -2, 0 polynomial x2+2x

2. K(2x2 +x-15)

3. a›0.
4. 2

5. k(x2-2√2x-7) where k is real. 6. 0.

7. 3/2 and -1/3 are the zeroes Verification to be done

8. x2-2√3x+2

LEVEL II
9. k= -2
10. k= 0
11. k= 0 or k=4 12. a= ½ , c = 5
13. Other Zeroes 2 and -3 14. K= -7
15. K=1 16. a= -16
LEVEL III
17. 58/9
18. x2-3x-28
19. 5
20. (i) 4 (ii) 1 (iii). -6

SELF EVALUATION QUESTIONS


21. (i) no zeroes (ii) 1 (iii). 2 (iv). 4
22. -2
23. Other zero 1 and P= -6
24. a=3
25. 20x2-9x+1
26. 2 zeroes, K= -2/3
Pair of Linear Equations in Two Variables
(Key Points)
 An equation of the form ax+by+c=0, where a,b,c are real nos. and a2+b2≠0is called a linear
equation in two variables x and y.
Ex: (i) x+5y+2=0 (ii)3 x - y = 1
2

 The general form for a pair of linear equations in two variables x and y is
a1x+b1y+c1=0
a 2 x + b 2 y + c2 = 0
Where a1,b1,c1,a2,b2,c2 are all real numbers and a1≠0,b1≠0,a2≠0,b2≠0. Example
x + 3y – 6 = 0
2x – 3y – 12=0
 Graphical representation of a pair of linear equations in two variables:
a1x+b1y+c1=0
a 2 x + b 2 y + c2 = 0

(i) Will represent intersecting lines if

I.e.unique solution. And these types of equations are called consistent pair of linear equations.

Ex: x – 2y = 0
3x + 4y – 20 = 0

Co-ordinates of the point of


intersection gives the
solution of the equations.

(ii) Willre present overlapping or coincident lines if

i.e. infinitely many solutions, consistent or dependent pair of linear equations

Ex:2x+3y–9=0 4x+6y–
18=0
The graph is Coincident lines,

(iii) will represent parallel lines if

i.e.no solution and called inconsistent pair of linear equations. Ex:


x + 2y – 4 =0
2x + 4y – 12 = 0

Parallel lines, no solution.

• Algebraic methods of solving a pair of linear equations:

(i) Substitution method


(ii) Elimination Method
(iii) Cross multiplication method (Deleted for session 2020-21)
Level -I

1. Find the value of „a‟ so that the point (2,9) lies on the line represented by ax-3y=5
2. Find the value of k so that the lines 2x–3y = 9 and kx - 9y = 18 will be parallel.
3. Find the value of k for which x + 2y = 5,3x + ky+15 = 0 is inconsistent
4. Check whether given pair of lines is consistent or not 5x–1=2y,y=–1 + 5 x
2 2

5. Determine the value of „a‟ if the system of linear equations 3x+2y-4=0and ax–y–3=0 will
represent intersecting lines.
6. Write any one equation of the line which is parallel to2x–3y=5
7. Find the point of intersection of line -3x+7y=3 with x-axis
8. For what value of k the following pair has infinite number of solutions.
(k-3)x+3y=k
K (x + y) =12
9. Write the condition so that a1x + b1y = c1 and a2x + b2y = c2 have unique solution.
Level -II

1. 5 pencils and 7 pens together cost Rs.50 whereas 7 pencils and 5 pens together cost Rs.46.Find the cost of
one pencil and that of one pen.

2. Solve the pair of linear equation:


3x – y = 3
7x + 2y = 20
3. Find the fraction which becomes to 2/3 when the numerator is increased by 2 and equal to 4/7 when the
denominator is increased by 4.
4. Solve the pair of linear equation
px + qy = p – q
q x – py = p + q
5. Solve the equation using the method of substitution:

3x – 5y = -1
x– y=-1
6. Solve the equations:
1 1
- = -1
2s y
1 1
+ =8; Where, x ≠ 0 , y ≠ 0
s 2y

7. Solve the equations by using the method of substitution :

x–y=7
5x + 12y =7

Level -III
1. Draw the graph of the equations

4x – y= 4
4x + y = 12
Determine the vertices of the triangle formed by the lines representing these equations and the x-axis.
Shade the triangular region so formed
2. Solve Graphically
x – y = –1
3x + 2y = 12.
Calculate the area bounded by these lines and the x- axis.

3. Solve for u& v.

4u–v=14uv
3u + 2v = 16uv where u≠0, v≠ 0
4. Ritu can row downstream 20 km in 2 hours, and upstream 4km in 2hours. Find her speed of rowing in still
water and the speed of the current. (HOTS)
5. In a , =3∠B=2(∠A+∠B) find the these angle. (HOTS)

6. 8 men and 12 boys can finish a piece of work in 10 days while 6 men and 8 boys can finish it in14 Days.

Find the time taken by 1 man alone and that by one boy alone to finish the work. (HOTS)

7. Find the value of K for which the system of linear equations 2x + 5y = 3, (k+1)x + 2(k+2)y = 2 K will
have infinite number of solutions. (HOTS)

SELF EVALUTION
1. Solve for x and y:
x + y = a+b
ax – by = a2 − b2

2. For what value of k will the equation x +5y – 7 = 0 and 4x + 20y + k = 0 represent coincident lines?

3. Solve graphically:

3x + y + 1 = 0
2x - 3y +8 = 0

4. The sum of digits of a two digit number is 9.If 27 is subtracted from the number, the digits
are reversed. Find the number.

5. Draw the graph of x+2y–7=0and 2x–y – 4 =0. Shade the area bounded by these lines and Y-axis.

6. Students of a class are made to stand in rows. If one student is extra in a row, there would be 2 rows
less. If one student is less in a row there would be 3 rows more. Find the number of the students in
the class.

7. Aman travels 370 km partly by train and remaining by car. If he covers 250 km by train and the
rest by the car it takes him 4hours, but if he travels 130 km by train and rest by car, he takes18
minutes longer. Find the speed of the train and that of the car.
8. Given linear equation 2x+3y – 8 = 0, write another linear equation such that the
geometrical representation of the pairs of lines are (i) intersecting lines, (ii)
Parallel Lines.
9. Solve for x and y.
(a-b)x +(a +b)y = a2 - 2ab – b2
(a+b)(x+y) = a2 + b2 (CBSE 2004, ’07C,’08)

10. The sum of two numbers is 8 and the sum of the its reciprocal is 8/15.Find the numbers.
(CBSE 2009)

11. The owner of a taxi cab company decides to run all the cars he has on CNG fuel instead
Of petrol/diesel. The car hire charges in city comprises of fixed charges together with the charge
for the Distance covered. For a journey of 12km, the charge paid Rs.89 and for a journey of 20km,
the charge paid is Rs.145.
i. What will a person have to pay for travelling a distance of 30km?
ii. Which concept has been used to find it?
iii. Which values of the owner have been depicted here?

12. Riya decides to use public transport to cover a distance of 300km.She travels this distance partly
by train and remaining by bus. She takes 4 hours if she travels 60 km by bus and the remaining by
train. If she travels100kmby bus and the remaining by train, she takes 10minutes more.
i. Finds peed of train and bus separately.
ii. Which concept has been used to solve the above problem?
iii. Which values of Riya have been depicted here?
ANSWER
LEVEL-I
Q1 a=16
Q2. k=6
Q3 .k=6
Q4. Consistent
Q5. a ≠ 3/ 2
Q6. (May be another solution also)
Q7. (-1,0)
Q8. k= 6
a1 b1
Q9. ≠
a2 b2

LEVEL-II

Q1. Cost of one pencil=Rs.3


Cost of one pen=Rs.5
Q2. X = 2, y = 3
Q3. 28/45
Q4. x=1, y = -1
Q5.

Q6.

Q7.

LEVEL-III
Q1. (2, 4)(1,0 )(3,0 )
Q2. x = 2, y = 3 and area = 7.5 unit 2
Q3. u = ½ , v = ¼
Q4.Speed of the rowing in still water = 6km/hr
Speed of the current=4km/hr.

Q5. ∠A=200, ∠B=400, ∠C=1200.


Q6: One man can finish work in140days. One boy can finish work in 280days.
Q7. K = 3
SELF EVALUATION
Q1. X=a y=b
Q2.K=-28
Q3.X= -1, y=2
Q4. 63
Q6. 60

30
Q7.Speed of the train=100km/h, speed of the car=80km/h
Q8.(i)4x-3y-8=0(may be another equation also)
(ii) 4x+6y+16 =0 (may be another equation also)
Q9. X= a+b, y= - 2ab/(a+b)
Q10. 3,5

Q11. (i) Rs.215,


(ii) A pair of linear equations in two variables has been used to find it.
(iii) Awareness of environment.
Q12. (i) The speed of the train = 80 km/h, the speed of the bus = 60km/h
(ii) A pair of linear equations in two variables has been used.
(iii) Controlling the pollution of the environment.

31 | P a g e
QUADRATIC EQUATIONS
KEY POINTS

1. The general form of a quadratic equation is ax2+bx+c=0, a≠o.a,b and c are real numbers.

2. A real number  is said to be a root of quadratic equation ax2+bx+c=0 where a≠0 if a2+b+c=0.The zeroes of the
quadratic polynomial ax2+bx+c and the roots of the corresponding quadratic equation a x2 + b x + c = 0 are the same.

3. Discriminant:-The expression b2 - 4ac is called Discriminant of the equation ax2+bx+c = 0 and is usually denoted
by D.Thus Discriminant D = b2 - 4ac.
4. Every quadratic equation has at most two roots which may be real, coincident or no real roots.

5. IF and are the roots of the equation ax2+bx+c=0 then

6. Sum of the roots

Product of roots

7. Forming quadratic equation, when the roots and are given by

8. Nature of roots of ax2+bx+c=0


a. If D >0, then roots are real and unequal.
b. If D = 0,then the equation has equal and real roots.
c. If D < 0,then the equation has no real roots.
d. If D > 0andD is a perfect square, then roots are rational and unequal.
e. If D >0andD is not a perfect square then roots are irrational and unequal.

9. Irrational roots always occur in conjugate pairs. If 2+√3 is one of the roots of the quadratic equation then other
root is 2-√3.

10. If a.b>0 the na>0and b>0 or a<0 and b<0 If a.b<0 the na>0 and b<0 or a<0 and b>0.

LEVEL-I
1. If 1 is a root of the equation x2 + kx - 5= 0, then find the value of K.
2 4

2. If D > 0,then write the roots of a quadratic equation ax2+bx+c = 0

3. Find the Discriminant of x2+ 5x+ 5 = 0.

4. Find the sum of roots of a quadratic equation x2 + 4x- 32 = 0

5. Find the product of the roots of the quadratic equation 2x2 + 7x – 4 = 0


6. Find the values of K for which the .equation 9x2 + 2kx +1 = 0 have real roots.
7. Find the Value of K if the equation x2- 2(k+1)x + k2 = 0 has equal roots.
8. For what value of k, x = a is a solution of equation x2 -(a+b)x+k=0?

9. Represent the situation in the form of Quadratic equation:

The Product of Rahman‟s age (in years) 5 years ago with his age 9 years later is 15.
10. Find the roots of x2- 3x -10 = 0
11. The product of two consecutive odd numbers is 483.Find the numbers.

LEVEL -II

1. If the x = 2 and x = 3 are roots of the equation 3x2 + 2 kx – 2M = 0, find the value of k and m.
s s+1 34
2. Solve the equation: + = , x ≠ -1, x ≠ 0.
s +1 s 15

3. Solve the equation 2x2 – 5 x + 3= 0 .


4. Using quadratic formula, solve the equation: p 2 x 2 + (p 2 – q 2)x – q 2 = 0
5. Find the Values of K for which equation 4x2- 2(k+1)x + ( k +4) = 0 has real & equal roots.

6. Find the roots of Quadratic equation 16x2–24x –1= 0 by using the quadratic formula.
7. Find the Discriminant of the Quadratic equation 2x2-4x+3=0and hence find the nature of its roots.

LEVEL – III

1. Solve for x :
2. Determine the Values of K for which equations x2+ 2kx + 64 = 0 and x2 - 8x + 2k = 0 will
both have real & equal roots..
3. Solve 3x2-23x-110 = 0
4. Solve the following equation for „x‟, 9x2-9(a+b)x+2a2+5ab+2b2 = 0
5. If the roots of the equation (a-b)x2+ (b-c)x + (c-a) = 0 are equal, prove that 2a = b+c.

Self-Evaluation
1. Find the value of „p‟ so that the equation 3x2 – 5x – 2p = 0has equal roots. Also find the roots.
3
2. The sum of two numbers is15.If the sum of their reciprocals is . Find the two numbers.
10

3. Find the quadratic equation whose roots are 2 + √3 and2 − √3.

4. Solve for x :
5. Divide 29 into two parts so that the sum of squares of the parts is 425.
6. Solve forx:9a2 – 6ax – (a2 – b2) = 0
7. If the equation (1 +m2)x2 – 2mcx – c2 – a2 = 0 has equal roots. Show that c2 = a2(1 + m2).

Answer
LEVEL-I
1. 2

–b ± ƒb 2–4ac
2. .
2a
3. 5
4. -4
5. - 2
6. -3≤ k ≤ 3
7. ± 1
8. K=ab
9. x2+4x-60 =0
10. -2 and 5
11. 21,23

LEVEL-II
1. m = 9 and k=-15/2
2. 3/2 or -5
3. 3/2 or 1
4. q2/p2 or -1
5. 5 , -3
3 ± √10
6.
4
7. -8, and roots are not real.

LEVEL-III

1. 7 , 9/2
2. 8
3. -10/3, 11
a+b 2(a+b)
4. ,
3 3

Self-Evaluation

1. 25/24
2. (10,5)or(5,10)
3. x2-4x +1=0
4. 6 , 7/2.
5. (13,16) or (16,13)
6. (a+b)/3 and (a- b)/3
KEY CONCEPT ARITHMETIC PROGRESSION

 An AP is a list of number in which difference of a term to its preceding term is always constant. The
constant is called common difference (d) of AP. d = an+1-an
 If ‟a‟ is the first term and „d‟ is the common difference of an AP, then the AP is a,a+d,a+2d,a+3d…..
 The nth term of an AP is denoted by an
an = a + (n-1) d where a = first term and d = common difference n= number of term
 nth term from the end = l - (n-1) d Where l = last term
 Various terms in an AP can be chosen in following manner.

No. of terms Terms Common difference


3 a-d ,a ,a+ d d
4 a-3d,a-d,a+d,a+3d 2d
5 a-2d,a-d,a,a+d,a+2d d

 Sum of first n natural number is n (n+1)/2.


 The sum of n terms of an AP with first term a and common difference d is
n
denoted by sn = [2a + ( n – 1) d]
2
 The sum of n terms of an AP with first term a and last term l.
s = n (a + l).
2
an = sn – sn-1
LEVEL-I
th
1. Write fourth term of an AP if its n term is 3n+2.
2. Find A.P which fifth terms are 5 and common difference is –3.
3. Determine the 10thterm from the end of the A.P:4,9,14 ... 254
4. Find whether 0 is a term of the A.P: 40,37,34,31………….
5. Write the value of x for which x+2,2x,2x+3 are three consecutive terms of an A.P
6. Find the sum of first 24 term of AP 5,8,11,14………..
7. Which term of the A.P 12,7,2,-3……is -98
th
8. The n term of an A.P is 3n+5 find its common difference.
9. Write the next term of an A.P. √2 , √18
10. If 4/5 ,a,2 three consecutive term of an A.P then find ‟a‟.

LEVEL- II
11. Find the middle term of A.P 6,13,20,... 216
12. The 6th term of an A.P is -10 and its 10th term is -26. Determine the15thterm of an A.P
13. The 8th term of an A.P is 0 prove that its 38t h term is triple its 18th term.
14. The sum of three numbers in A.P is 21 and their product is 231 find the numbers.
15. Find the sum of 25th term of an AP which nth term is given by tn=(7-3n)
16. Find the sum of all two digit odd positive numbers
17. Find the sum of three digits numbers which are divisible by 11
18. The sum of first 6 term of A.P is 42.The ratio its 10th term to 30th term
is1:3. Calculate the first and 13th term of the A.P
19. How many term of the A.P: 17,15,13,…must be added to get the sum 72?
Explain the double answer.
20. The sums of n,2n,3n term of an A.P are S1,S2,and S3 respectively.Prove that S3=3(S2-S1)
Deleted

Deleted

37
ANSWER
LEVEL-I
1. 14
2. 17
3. 209
4. no
5. 5
6. 948
rd
7. 23 term
8. 3
9. 5√2
10. 7/5

LEVEL- II
11. 111
12. -46
14. (-3,7,11)
13. -800
14. 2475
15. 44550
16. 2, 26
17. 6,12

LEVEL- III
21.

22. n=0

23. ..
24. 5 years

38
TRIANGLES

Key Points
Similar Figures: Two figures having similar shapes (size may or may not same), called Similar
figures.

Examples :(a) & (b) & (c) &


A pair of Circles A pair of squares A pair of Equilateral Triangles

 Pairs of all regular polygons, containing equal number of sides are examples of Similar
Figures.
 SimilarTriangles:Two Triangles are said to be similar if
(a) Their corresponding angles are equal (also called Equiangular Triangles)
(b) Ratio of their corresponding sides are equal/proportional
 All congruent figures are similar but similar figures may/may not congruent
 Conditions for similarity of two Triangles
(a) AAA criterion/A-A corollary
(b) SAS similarity criterion
(c) SSS similarity criterion(where „S‟ stands for ratio of corresponding sides of two
Triangles)

Important Theorems of the topic Triangles

(a) Basic Proportionality Theorem(B.P.T.)/Thales Theorem


(b) Converse of B.P.T.
(c) Pythagoras Theorem
LEVEL I
(1) In the figure XY ∥ QR,PQ/XQ=7/3 and PR=6.3cm then find YR
(2) If ∆ABC ~ ∆DEF and their areas be 64cm 2 & 121cm2 respectively ,then find BC if EF=15.4cm(Deleted)
(3) ABC is an isosceles ∆,right angled at C then prove that AB 2=2AC2
(4) If ∆ABC~∆DEF,∠ A=46 ∠ E = 62 then the measure of ∠ C = 72 .Is it true? Give reason.
0, 0 0

(5) The ratio of the corresponding sides of two similar triangles is 16:25 then find the ratio
of their perimeters.
(6) A man goes 24km in due east and then He goes 10 km in due north. How far is he from
the starting point?
(7) The length of the diagonal so far hombus is 16cm & 12cm respectively then find the perimeter of the
rhombus.
(8) In the figure LM|| CB and LN|| CD then prove that AM/AB=AN/AD

(9) Which one is the sides of a right angled triangles among the following (a)6cm,8cm&11cm
(b) 3cm,4cm & 6cm (c) 5cm , 12cm & 13cm

Level II
(1) In the figure ABD is a triangle right angled at A and AC is perpendicular to BD then show that
AC2=BC x DC

(2) Two poles of height 10m & 15m stand vertically on a plane ground.If the distance between
Their feet is 5 √3m ,then find the distance between their tops.
(3) D &E are the points on the sides AB & AC of ∆ABC ,as shown in the figure .If∠ B=∠ AED then show
that ∆ABC~∆AED
4) In the adjoining figure AB || DC and diagonal AC & BD intersect at point O. If AO=(3x-1)cm,

B=(2x+1)cm ,OC=(5x-3)cm and OD=(6x-5)cm then find the value of x.


5) In the figure D & E trisect BC. Prove that 8AE2=3AC2+5AD2

6) In the figure OA/OC=OD/OB then prove that∠A=∠C

7) Using converse of B.P.T. Prove that the line joining the mid points of any two sides of a triangle is
parallel to the third side of the triangle.

8) In the given figure ∆ABC & ∆DBC are on the same base BC .If AD intersect BC at O then
prove that ar(∆ABC)/ar(∆DBC)=AO/DO

Level III

(1) A point O is in the interior of a rectangle ABCD, is joined with each of the vertices A,B,C&D.
Prove that OA2 +OC2 = OB2+OD2

(2) In an equilateral triangle ABCD is a point on the base BC such that BD=(1/3)BC ,then show that
9AD2=7AB2
(3) Prove that in a rhombus, sum of squares of the sides is equal to the sum of the squares of its
diagonals

(4) In the adjoining figure ABCD is a parallelogram. Through the mid -point M of the side CD ,a line is
drawn which cuts diagonal AC at Land AD produced at E. Prove that EL=2BL

(5) ABC & DBC are two triangle so the same base BC and on the same side of BC with
∠A=∠D=900.If CA &BD meet each other at E then show that AE x EC = BE x ED

(6) In the figure AB || DE and BD || EF. Prove that DC2= CF x AC

Self-Evaluation Questions including Board Questions

(1) Find the value of x for which DE||BC in the adjoining figure

(2) In an equilateral triangle prove that three times the square of one side is equal to four times the
square of one of its altitude.
(3) The perpendicular from A on the side BC of a triangle ABC intersect BC at D such that DB=3CD.
Prove that 2AB2= 2AC2+ BC2
(4) In the adjoining figure P is the mid point of BC and Q is the mid point of AP .If BQ when
produced meets AC at R ,then prove that RA=(1/3)CA
(5) BL and CM are medians of triangle ABC, right angled at A then prove that 4(BL2 + CM2) = 5BC2

(6) In ∆ABC if AB = 6√3cm , AC = 12cm and BC = 6cm then show that ∠B=900

(7) In the adjoining figure ∠QRP=900, ∠PMR=900,QR = 26cm ,PM=8cm and MR=6cm then find the area of
∆PQR

(8) If the ratio of the corresponding sides of two similar triangles is 2:3 then find the
ratio of their corresponding altitudes. (DELETED)
(9) In the adjoining figure ABC is a ∆ right angle d at C. P & Q are the points on the sides CA &CB
respectively which divides these sides in the ratio2:1,then prove that 9(AQ2+BP2)=13 AB2

(10) The adjoining figure AB || PQ || CD,AB =x unit, CD =y unit & PQ = z unit then prove that 1/x + 1/y = 1/z

(11) State and prove Pythagoras theorem .Using this theorem find the distance between the tops of two vertical
poles of height 12m &18m respectively fixed at a distance of 8m apart from each other.
(12) In the adjoining figure DEFG is a square &∠BAC=900 then prove that
(a) ∆AGF ~ ∆DBG (b) ∆ AGF ~ ∆EFC ( c) ∆ DBG ~∆EFC (d) DE2 = BD XEC
45
25

45
46
48
49
50
51
Deleted

52
LEVEL - I

1. Find the distance between the points P (7,5) and Q(2,5).


2. If P (α/3 ,4) is the midpoint of the line segment joining the points Q(-6,5) and R( -2,3),then find the
value of α.
3.A line intersects y–axis and x-axis at the points P and Q respectively. If (2,-5) is the midpoint of PQ, then
find the coordinates of P and Q respectively.
4. If the distance between the points (4,p) &(1,0) is 5, then find the value of p.
5. If the point A(1,2), B(0,0) and C(a ,b) are collinear, then find the relation between a and b.

6. Find the ratio in which the y-axis divides the segment joining (-3,6) and(12,-3).
7. Find the coordinates of a point A, where AB is diameter of a circle whose Centre is(2,-3)and B is(1,4)
8. Find the centroid of triangle whose vertices are (3,-7), (-8,6) and (5,10).

LEVEL - II

1. If A (-2,4),B(0,0),C(4,2) are the vertices of a ∆ABC, then find the length of median through the vertex A.
2. Find the value of x for which the distance between the points P(4,-5) and Q(12,x) is 10 units.
3. If the points A (4,3) and B (x,5) are on the circle with Centre O(2,3) then find the value of x.
4. What is the distance between the point A(c,0) and B(0,-c)?
5. For what value of p, are the points (-3,9),(2,p) and(4,-5) collinear?
6. Show that the points (3,2),(0,5),(-3,2) and (0,-1) are the vertices of a square.
7. Point P divides the line segment joining the points A (2,1) and B (5,-8) such that
AP: AB=1:3 If P lies on the line2x-y+k=0, then find the value of k.
8. Find the relation between x and y if the points(2,1),(x , y) and (7,5) are collinear
LEVEL - III
1. Find the ratio in which the line2x+3y=10 divides the line segment joining the points (1,2) and (2,3).
2. Prove that(4,-1),(6,0),(7,2)&(5,1) are the vertices of a rhombus .Is it a square?
3. Find the coordinates of the points of trisection of the line segment joining the points
A(7,-2) and B is(1,5)
4. Determine the ratio in which the point P(a,-2) divides the line joining of points (-4,3) and B(2,-4).Also
find the value of a.
5. If the point C(-1,2) divides internally the line segment joining A (2,5) and in the ratio 3:4. Find the Co-
ordinates of B.
6. Show that points (1,1), (4,4) ,(4,8)and (1,5) are the vertices of a parallelogram.
7. Find the value of p, for which the points (-1,3),(2,p) & (5,-1) are collinear
8. If the points(-1,3),(1,-1) and (5,1) are the vertices of a triangle .Find the length of the median through the
first vertex.

SELF EVALUATION

1. Find the center of a circle passing through the points (6,-6), (3,-7) and (3,3).
2. If the distance between the points (3,0) and (0,y) is 5 units and y is positive ,what is the value of y?
3. If the points(x ,y),(-5,-2)and(3,-5) are collinear r, then prove that 3x+8y+31=0.
4. Find the ratio in which the Y-axis divides the line segment joining the points (5,-6) and (-1,-4). Also find
the coordinates of the point of division.

5. By distance formula, show that the points (1,-1), (5,2) and (9,5) are collinear.

6. Show that the three points (a,a) ,(-a,-a) &(-a√3 , a√3) are the vertices of an equilateral triangle.

Board Questions

Q: 1) Find the value of k, if the point P (2, 4) is equidistant from the points (5, k) and (k, 7).
(CBSE: 2012)
Q:2)If the point A(0,2)is equidistant from the points B (3,p) and C(p,5),find p .Also find the length of AB.
(CBSE: 2014)
Q:3)Find the ratio in which the point P (x,2) divides the line-segments joining the points A (12,5) and
B(4,- 3).Also ,find the value of x. (CBSE: 2014)
Q:4)If the points A (-2, 1),B (a, b) and C(4,-1) are collinear and a-b=1.Find the value of a
and b. (CBSE: 2014)
Q: 5) In what ratio does the point (-4, 6) divides the line segment joining the points A (-6, 10) &B (3,-8)
(CBSE: 2012)
ANSWERKEY
LEVEL-I
1. 5
2. -12
3. (0,-10) and(4,0)
4. ±4
5. 2a=b
6. ¼
7. (3,-10)
8. (0,3)
LEVEL-II
1. 5 units
2. 1, - 11
3. 2
4. √2 c
5. -1
6. Proof
7. K=-8
8. 4x - 5y - 3=0

LEVEL-III
1. 2:3
2. Proof
3.( 5,1/3) (3,8/3)
4. a=2/7
5. B(-5,-2)
6. Proof
7. p=1
8. 5
SELF EVALUATION
1. (3,-2)
2. 4
3. Proof
4. 5:1, (0,-13/3)
5. Proof
6. Proof
BOARD QUESTIONS
1.K=3
2. P=1, AB=√10
3. 3:5, x=9
4. a=1, b=0
5.2/7

53
54
4. Trigonometric Identity -- An equation involving trigonometric ratios of an angle is called a
trigonometric identity if it is true for all values of the angle.
Important trigonometric identities:
(i) sin2 + cos2 = 1
(ii) 1+ tan2 = sec2
(iii) 1 + cot2 = cosec2

5. Trigonometric Ratios of some specific angles

Name/angles 00  450  900


sin  0  1/2 3/2 1
cos  1 3/2 1/2 /2 0
tan  0 1/3 1 3 Not defined
cot  Not 3 1 1/3 0
defined
sec  1 2/3 2 2 Not defined
cosec  Not 2 2 2/3 1
defined
Level – I (These questions can be taken as MCQ/FILL UP/TRUE AND FALSE i.e.1 mark each)

1. If sin θ = 1/3 , then find the value of 2 cot2 θ +2 . (18)

2. Evaluate sec2 60° + sec 0° (5)

3. Find the value of (sin θ+ cos θ)²+(cos θ-sin θ)² (2)

4. If tan θ= ¾, then find the value of cos² θ-sin²θ (7/25)

5. If sec θ + tan θ= p, then find the value of sec θ – tan θ (1/p)

6. Change sec⁴ θ-sec²θ in terms of tanθ. (tan2θ+tan4θ)


7. Prove that
(CBSE2009)

8. In a triangle ABC, it is given that ∠C=90˚and tan A=1/√3,find the value of (sinAcosB+cosAsinB)
(1) (CBSE2008)
9. If 0° ≤ x ≤ 90°and 2sin²x = 1/2,then find the value of x (300)

10. Simplify (secθ +tanθ)(1-sinθ) ( cosθ)


11. Prove that cosA /(1-sinA)+cosA /(1+sinA)=2secA

Ans 1-12/25

4. Prove that .+ =2 cosec A


5. Prove that (sinθ + cosecθ)²+(cosθ+secθ)²=7+tan²θ+cot²θ. (CBSE 2008,2009C)

6. Evaluate: (1+cot θ- cosec θ)(1+tan θ +cosec θ) = 2

7. Find the value of sin30˚geometrically. (Not from examination point of view)


8. If tan (A-B) =√3, and sin (A+B) =1, then find A and B. Ans: (A=75, B=15)
9. If θ is an acute angle and sin θ=cosθ, find the value of 3 tan²θ +2 sin²θ –1. Ans: 3

10. If cos θ+ sin θ = 1 and sinθ – cos θ=1, prove that x²/a²+y²/b²=2.

---
11. Prove that =tan θ.
Level – III
1. If = m and = n, show that (m²+n²) cos²β=n². (CBSE2012)

2. Prove that tan²θ+cot²θ+2=cosec²θ sec²θ


3. If cosθ+sinθ=√2cosθ, then show that (cosθ -sinθ) =√2sinθ.

4. Prove that (sinθ+secθ) ²+ (cosθ+cosecθ)² = (1+secθcosecθ)².


5. Prove that sin θ/ (1-cosθ) +tan θ/(1+cosθ)=secθcosecθ+cotθ.

6. If x=a sin θ and y=b tan θ. Prove that a2/x2–b2/y2=1.


7. Prove that sin6θ+ cos6θ=1-3sin2θ cos2θ.
8. Prove that (sec θ+ tan θ–1)/ (tan θ–secθ+1) = cos θ/(1–sin θ). (CBSE 1997, 2002, 2007)

9. Prove that (1+cotθ-cosecθ)(1+tanθ+secθ)=2 (CBSE2005,07,08)

10. If x= p sec θ + q tan θ and y= p tan θ + q sec θ , then prove that x2-y2 = p2 – q2

2
11. If sinθ+cosθ = m and secθ+cosecθ=n, then prove that n (m –1) =2m.

Self-Evaluation

1. If a cosθ+ b sinθ = c, then prove that a sinθ–b cosθ= □


2. If A,B,C are interior angle of triangle ABC, show thatcosec² ( ) - tan² =1.

3. If sinθ+sin²θ+sin³θ=1, prove that cos⁶ θ–4cos⁴ θ+8cos²θ=4.

4. If tan A=n tan B, sin A=m sin B, prove that cos²A= (m²-1)/(n²-1).

5. If secθ + tanθ=p, prove that sinθ = (p²-1)/(p²+1).

6. Prove that - = - .

7. Prove that: sin2θ tanθ + cos2θ cotθ + 2sinθcosθ = tanθ+cotθ = secθ cosecθ

8. Prove that =_ _ .

9. Prove that(1+cosθ+sinθ)/(1+cosθ –sinθ)=(1+sinθ)/cosθ


SOMEAPPLICATIONS OF TRIGONOMETRY

HEIGHT AND DISTANCES

KEY POINTS

Line of sight
Line segment joining the object to the eye
of the observer is called the line of sight.

Angle of elevation
When an observer sees an object situated in
upward direction, the angle formed by line of
sight with horizontal line is called angle of
elevation.

Angle of depression
When an observer sees an object situated in
downward direction the angle formed by line of
sight with horizontal line is called angle of
depression.
Level – I (These questions can be taken as MCQ/FILL UP/TRUE AND FALSEi.e.1 mark each)

1. A pole 3m high casts a shadow m long on the ground, then find the sun‟s elevation?

2. If length of the shadow and height of a tower are in the ratio1:1.Then find the angle of elevation.

3. An observer 1.5m tall is 20. 5 metres away from a tower 22m high. Determine the angle
of elevation of the top of the tower from the eye of the observer.
4. A ladder 15m long just reaches the top of vertical wall. If the ladder makes an angle 60 0 with the wall,
find the height of the wall.
5. In a rectangle ABCD, AB=20cm BAC= 60 0 then find the length of the side AD.
6. Find the angle of elevation of the sun‟s altitude when the height of the shadow of a vertical pole is
equal to its height:
7. From a point 20 m away from the foot of a tower, the angle of elevation of top of the tower is 30°, find
the height of the tower.
8. In the adjacent figure, what are the angles of elevation and depression of the top and bottom of
a pole from the top of a tower h (m) high: Q O
0 0
Ans 45 , 60
300
450
A M

B L

LEVEL –II
9. The length of the shadow of a pillar is times its height. Find the angle of elevation of the
source of light.

10. A vertical pole10m long casts a shadow m long. At the same time tower casts a shadow 90m
long. Determine the height of the tower.
11. A ladder 50m long just reaches the top of a vertical wall. If the ladder makes an angle of 600 with the
wall, find the height of the wall.
12. Two poles of height 6m and 11m stands vertically on the ground. If the distance between their feet
is12m. Find the distance between their tops.
13. The shadow of tower, when the angle of elevation of the sun is 45 o is found to be 10m longer than
when it is 60o.Find the height of the tower.
LEVEL –III

14. The angle of depression of the top and bottom of a tower as seen from the top of a 100 m high cliff
are 300 and 600 respectively. Find the height of the tower.
15. From a window (9m above ground) of a house in a street, the angles of elevation and depression of the
top and foot of another house on the opposite side of the street are 300 and 600 respectively.
Find the height of the opposite house and width of the street.
16. From the top of a hill, the angle of depression of two consecutive kilometer stones due east are found to
be 300 and 450.Find the height of the hill.
17. Two poles of equal heights are standing opposite each other on either side of the road, which is 80m
wide. From a point between the road the angles of elevation of the top of the poles are 600 and
300. Find the heights of pole and the distance of the point from the poles.

18. The angle of elevation of a jetfighter from a point A on the ground is 600.After a flight of 15
seconds, the angle of elevation changes to 300. If the jet is flying at a speed of 720 km/hr, find
the constant height at which the jet is flying.

19. A window in a building is at a height of 10 m above the ground. The angle of depression of a point P on
the ground from the window is 300.The angle of elevation of the top of the building from the point P is
600. Find the height of the building.
20. A boy, whose eye level is 1.3 m from the ground, spots a balloon moving with the wind in a
horizontal line at same height from the ground. The angle of elevation of the balloon from the
eyes of the boy at any instant is 600.After 2 seconds, the angle of elevation reduces to 300 if the
speed of the wind at that moment is m/ then find the height of the balloon from the ground.

21. A man on the deck on a ship14m above water level observes that the angle of elevation of the top of
a cliff is 600 and the angle of depression of the base of the cliff is 300.Calculate the distance of the
cliff from the ship and the height of the cliff.
22. A tower is 50m high. It‟s shadow is x m shorter when the sun‟s altitude is 45o than when it is 30o. Find
x correct to the nearest 10.
SELF EVALUATION / HOTS
23. An airplane when flying at a height of 3125m from the ground passes vertically below another Plane at
an instant when the angle of elevation of the two planes from the same point on the ground are 30°
and 60° respectively. Find the distance between the two planes at that instant.
24. From the top of a building 60m high, the angels of depression of the top and bottom of a vertical lamp
post are observed to be 30° and 60°respectively. Find
[i] horizontal distance between the building and the lamp post
[ii] height of the lamp post.
25. A vertical tower stands on a horizontal plane and is surmounted by a vertical flagstaff of height
„h‟ m. At a point on the plane, the angles of elevation of the bottom and the top of the flag staff are
α and β respectively. Prove that the height of the tower is (h tan α) / (tanβ -tanα).
26. The angle of elevation of a cloud from a point 60m above a lake is 30° and the angle of depression of the
reflection of the cloud in the lake is 60°. find the height of the cloud from the surface of the lake.
27. A round balloon of radius „r‟ subtends an angle α at the eye of the observer whose angle of elevation of
centre is β. Prove that the height of the centre of the balloon is (r sinβ cosec α/2).
28. A person standing on the bank of a river observes that the angle of elevation of top of building of an
organization working for conservation of wildlife. Standing on the opposite bank is 60°.When he moves
40 m away from the bank he finds the angle of elevation to be 30°.Find the height of the building and
width of the river.

ANSWERS
Level 1:-
1)600 (2) 450 (3) 450 (4) 7.5 m (5) 20 m (6) 450 (7) 20 / √3 m (8) 450,600
Level 2:-
(9)300 (11) 25√3 m (12) 13 m (13) 5(√3 + 1) m
(10)30√3 m

Level 3:-
(14) 66.67m (15) 3m ,3 m (16)1.37km (17) 20 m, 20m and 16m (18) 500 m (19) 30m
(20) 87m (21)14 m and 56m (22) 36.6m

Self Evaluation
(23) 6250m (24) 20 , 40m (26) 120m (28) 20 , 20m
CIRCLES
1. Circle: A circle is a collection of all points in a plane which are at a constant distance (radius)
from a fixed point (centre).

2. Secant & Tangent to a Circle: In fig.1 the line PQ and the circle have no common point. Line
PQ is called non-intersecting.Infig.2 line PQ a secant to a circle.Infig.3, there is only 1 point A,
which is common to the line PQ and the circle .The line is called a tangent to the circle.

3. Tangent to a Circle:
It is a line that intersects the circle at only one point. There is only one tangent at a point of the
circle. The tangent to a circle is a special case of the secant, when the two end points of its
corresponding chord coincide.

4. Theorems:
1. The tangent at any point of a circle is perpendicular to the radius through the point of
contact.
2. The lengths of tangents drawn from an external point to a circle are equal.

5. Number of tangents from a point on a circle-


(i) There is no tangent to a circle passing through a point lying inside the circle.
(ii) There is one and only one tangent to a circle passing through a point lying on the circle.
(iii) There are exactly two tangent to a circle through a point lying outside the circle.
LEVEL I
(These questions can be taken in the form of MCQ/FILL UP/TRUE AND FALSE i.e.1 mark
each)

1. In the given fig. O is the centre of the circle and PQ is tangent then POQ + QPO is equal to

2. If PQ is a tangent to a circle of radius 5cm and PQ = 12 cm, Q is point of contact, then OP is


3. In the given fig. PQ and PR are tangents to the circle , ∠QOP= 70°, then ∠QPR is equal to

4. In the given fig. QS is a tangent to the circle, OS = 8 cm, OQ = 6 cm then the length of QS is

5. In the given fig PQ is tangent to outer circle and PR is tangent to inner circle. If PQ = 4 cm, OQ = 3 cm
and OR = 2 cm then the length of PR is

P
6. In the given fig. P, Q and R are the points of contact. If AB = 4 cm, BP = 2 cm then the perimeter of
ABC is

7. The distance between two tangent parallel to each other to a circle is 12cm. The radius of circle is

8. The chord of a circle of radius 10cm subtends a right angle at its centre. Find the length of the chord.
9. How many tangents can a circle have?

10. How many tangents can be drawn from a given exterior point to a circle?

LEVEL - II
11. Two concentric circles of radii a & b (a>b) are given. Find the length of the chord of the larger circle
which touches the smaller circle
12. From a point P outside the circle with centre O, tangents PA and PB are drawn to the circle. Prove that
OP is the right bisector of the line segment AB.
13. A circles is inscribed in a triangle ABC, touching BC, CA and AB at P, Q and R respectively. If AB= 10
cm AQ =7cm CQ =5 cm. Find BC.

14. A Quadrilateral ABCD is drawn to circumscribe a circle, as shown in the figure. Prove that AB+CD
= AD + BC
15. Two concentric circles are of radii 7 cm and r cm respectively, where r>7. A chord of the larger circle
of length 46 cm, touches the smaller circle. Find the value of „r‟.

16. Prove that the tangent at any point of a circle is perpendicular to the radius through the point of
contact.

LEVEL - III
17. Prove that the length of tangents drawn from an external point to a circle are equal.

18. Prove that the tangents at the extremities of any chord of a circle, make equal angle with the chord.
19. PA and PB are tangents to the circle with the centre O from an external point P, touching the circle at A
and B respectively. Show that the quadrilateral AOBP is cyclic.
20. Prove that the parallelogram circumscribing a circle is a rhombus.
21. In the given figure, XY and X‟Y‟ are two parallel tangents to a circle with centre O and another tangent
AB with point of contact C intersects XY at A and X‟Y‟ at B. Prove that
AOB=900.

Q.22 Two roads starting from P are touching a circular path at A and B. Sarita runs from P to A, 20km and A to
O, 15km and Reeta runs from P to O directly.
(a) Find the distance covered by Reeta.
(b) Who will win the race?
SELF EVALUATION

1. Draw a circle and two lines parallel to a given line such that one is a tangent and the other, a secant
to the circle.
2. Prove that perpendicular at the point of contact to the tangent to a circle passes through the
centre.

3. Prove that the angle between the two tangents drawn from an external point to a circle is
supplementary to the angle subtended by the lines segment joining the points of contact at
the centre.
4. The length of a tangent from a point A at a distance 5cm from the centre of the circle is 4cm.Find the
radius of the circle. Ans:12cm
5. Two concentric circles are of radii 6.5 cm and 2.5 cm.Find the length of the chord of larger circle
which touches the smaller circle. Ans:3cm
6. From a point P, 10cm away from the centre of the circle, a tangent PT of length 8cm is drawn. Find
the radius of the circle. Ans:6cm
Level III

17. Correct construction


Figure
Proof
18. Correct construction
Figure
Proof

19.
Quad. OAPB,
∠ AOB+∠ OAP+∠ APB+∠ OBP=3600Or,
∠ AOB+900+∠ APB+900=3600
Or, ∠ AOB+∠ APB+1800=3600Or,
∠ AOB+∠ APB=1800
Hence, quad. OAPB is cyclic.
20.
AP=AS ..................... (i) [Tangents from A]
BP=BQ ................... (ii)[Tangents from B]
CR=CQ ...................... (iii)[Tangents from c]
DR=DS ..................... (iv)[Tangents from D]
Now, AB+CB=AP+BP+CR+DR
= AS + BQ + CQ + DS [From (i), (ii), (iii), (iv)]
= (AS + DS ) + (BQ + CQ)
=AD+BC Or,
AB+CD=AD+BC Or, 2AB =2AD
Or, AB = AD
Hence, AB = BC = CD = AD
Hence, ABCD is a rhombus.
21. In quad. APQB
∠ APO+∠ BQO+∠ QBC+∠ PAC=3600Or,90
0+900+∠ QBC+∠ PAC=3600
Or,∠ QBC+∠ PAC=180 0 ................................................... (i) We
have,∠ CAO=½∠ PAC And
∠ CBO = ½ ∠ QBC
Now, ∠ CAO + ∠ CBO = ½ (∠ PAC + ∠ QBC )
= ½ X 1800 (from eq. i)
=90 0 ............................................ (ii)
In triangle AOB,
∠ CAO + ∠ AOB + ∠ CBO = 1800
Or, ∠ AOB + 900 = 1800 (from eq. ii)
Or,∠ AOB=900
22.(i)

In triangle OAP,
76
OP2=OA2+AP2 (By
Pythagoras
Theorem)
Or,OP2=(15)2+(20)2
Or, OP2= 625
Or, OP=25km
(ii) Distance covered by Reeta = 25 km Distance
covered by Sarita=20km+15km= 35 km
So, Rita will
win the race.

CONSTRUCTIONS
1. Division of a line segment in the given ratio.

2. Construction of triangles:-

a. When three sides are given.


b. When two sides and included angle given.
c. When two angles and one side given.
d. Construction of a right angled triangle.

3. Construction of triangle similar to a given triangle as per given scale factor.(Deleted)

4. Construction of tangents to a circle.

EXPECTED LEARNING OUTCOMES

1. Correct use of Mathematical instruments.


2. Drawing a line segment and an angle as per the given data.
3. To divide the given line segment in the given ratio accurately.
4. Neatness and accuracy in drawing.
5. The concept of similar triangles.
6. To Construct a triangle as per the conditions given.
7. To construct tangents to a circle from an external point given.

76
LEVEL – I
1. Draw a line segment AB=8cm and divide it in the ratio 4:3.

2. Divide a line segment of 7cm internally in the ratio2:3.

3. Draw a circle of radius 4cm.Take a point P on it. Draw tangent to the given circle at P.

4. Construct an isosceles triangle whose base is7.5cm and altitude is 4.2cm.

5. Draw a line segment of length 9cm.and divide it in seven equal parts.

LEVEL –II

1. Draw a pair of tangents to a circle of radius 4cm, which are inclined to each other at an angle of 600.
(CBSE2013)

2. Draw a circle of radius 5cm. From a point 8cm away from its centre construct the pair of tangents to the circle and
measure their lengths.
3. Draw a line segment AB=7.5cm and locate a point P on AB such that AP=3/7AB.Give justification of the
construction.

LEVEL-III
1. Draw a circle with centre O and radius 3.5cm. Take a horizontal diameter. Extend it to both sides to point P and Q
such that OP=OQ=7cm.Draw tangents PA and QB, one above the diameter and the other below the diameter. Is
PA||BQ.
2. Draw a line segment AB of length 8cm.Taking A as centre, draw a circle of radius 4cm and taking B as centre,
draw another circle of radius 3cm.Construct tangents to each circle from the centre of the other circle.
3. Draw a line segment AB of length 7cm.Using ruler and compasses, find a point P on AB such that AP/AB= 3/5.
(CBSE2011)

4. Construct a tangent to a circle of radius 3.5 from a point on the concentric circle of radius6.5cmand measure
its length. Also, verify the measurement by actual calculation.
Self-Evaluation
1. Draw a line segment of length 7cm. Find a point P on it which divides it in the ratio 3:5.

2. Two line segments AB and AC include an angle of 600 where AB=5cmand AC=7CM.Locate points P and Q on AB
and AC respectively such that AP=3/4AB and AQ=1/4AC.Join P and Q and measure the length PQ.

3. Draw a pair of tangents to a circle of radius 4.5cm, which are inclined to each other at an angle of 450.
4. Draw a line segment AB of length 7cm.Taking A as centre, draw a circle of radius 3cm and taking B as centre
another circle of radius 2.5cm.Construct tangents to each circle from the centre of the other circle.

5. Two trees are to be planted at two positions A and B in the middle of a park and the third tree is to be planted at
a position C in such a way that AC: BC=3:4.How it can be done?

6. Draw a circle of radius 5cm.Draw tangents from the end points of its diameter. What do you observe?
AREAS RELATED TO CIRCLES
KEY POINTS

1. Circle: The set of points which are at a constant distance from a fixed point in a plane is called a circle.

2.Circumference: The perimeter of a circle is called its circumference.


3.Secant: A line which intersects a circle at two points is called secant of the circle.
4.Arc: A continuous piece of circle is called an arc of the circle.
5.Central angle :- An angle subtended by an arc at the center of a circle is called its central angle.
6.Semi-Circle:-A diameter divides a circle in to two equal arcs. Each of these two arcs is called a semi- circle.
7.Segment :- A segment of a circle is the region bounded by an arc and a chord, of a circle.
8.Sector of a circle: The region enclosed by an arc of a circle and its two bounding radii is called a sector of the
circle.
9. Quadrant :- One fourth of a circle/circular disc is called a quadrant. The central angle of a quadrant is 900.

S.N NAME FIGURE PERIMETER AREA


1. Circle πr²

2πr or πd

πr + 2r 1/2 nr²

Semi- circle
2.

Ring (Shaded region) 2π(r+R) π ( R2 − r²)


3.

L+2r=  πr+2r  πr²


4. Sector of a circle 360
180

5.
Area of Segment  πr+2r sin  θ
πr²- 1r²sinθ
of a circle 180 2 360 2
a. Length of an arc AB= θ 2 π
360

a. Area of major segment=Area of a circle–Area of minor segment

b. Distance moved by a wheel in 1 rotation=circumference of the wheel

c. Number of rotation in 1 minute=Distance moved in 1 minute/circumference

LEVEL-I

1. If the perimeter of a circle is equal to that of square, then the ratio of their areas is

i. 22/7
ii. 14/11
iii. 7/22
iv. 11/14

2. The area of the square that can be inscribed in a circle of radius 8cm is
i. 256 cm2
ii. 128 cm2
iii. 64√2 cm2
iv. 64 cm2

3. Area of sector of a circle of radius 36 cm is 54π cm2.Find the length of the corresponding arc of
the circle is
i. 6π cm
ii. 3π cm
iii. 5π cm
iv. 8π cm
4. A wheel has diameter 84cm.The number of complete revolution it will take to cover 792m is.
i. 100
ii. 150
iii. 200
iv. 300
5. The length of an arc of a circle with radius 12cm is 10π cm. The central angle of this arc is.
i. 1200
ii. 60
iii. 750
iv. 1500

6. The area of a circle whose circumference π cm


is i. 11/2cm2
ii. π/4cm2
iii. π/2cm2
iv. None of these

7. In figure „o‟ is the centre of a circle. The area of sector OAB is 5/18 of the area of the circle find x.

8. If the diameter of a semicircular protractor is 14cm, then find its perimeter.


9. The diameter of a cycle wheel is 21cm.How many revolutions will it make to travel 1.98 km?
10. The length of the minute hand of a clock is 14cm.Find the area swept by the minute hand in 5 minutes.

LEVEL – II

1. Find the area of the shaded region in the figure if AC=24cm,BC=10cm and 'o' is the center of the circle
(use n = 3.14)

B C

2. The inner circumference of a circular track is 440m.The track is 14m wide. Find the diameter of the outer circle of
the track. [Take n=22/7]
3. Find the area of the shaded region.

4. A copper wire when bent in the form of a square encloses an area of 121cm2.If the same wire is bent in to the form
of a circle, find the area of the circle(use π =22/7)

5. A wire is looped in the form of a circle of radius 28cm.It is rebent in to a square form. Determine the side of the
square(use π =22/7)

LEVEL-III
1. Three horses are tethered with 7m long ropes at the three corners of a triangular field having sides 20m, 34m
42m.Find the area of the plot.
i. Grazed by horses
ii. Remains un grazed by horses

2. Calculate the area of shaded region in given figure where ABCD is square of side16cm.

3. ABC is a quadrant of circle of radius14cm and a semi-circle is drawn with BC as diameter. Find the area of
Shaded region.
4. The length of a minor arc is 2/9 of the circumference of the circle. Write the measure of the angle
subtended by the arc at the centre of the circle.

5. The area of an equilateral triangle is 49√3cm2. Taking each angular point as centre, circle is drawn with radius
equal to half the length of the side of the triangle. Find the area of triangle not included in the
circles. [Take √3=1.73]

SELF EVALUATION

1. Two circles touch externally the sum of the areas is 130π cm2 and distance between there centre is 14cm.
Find the radius of circle.
2. Two circle touch internally. The sum of their areas is 116π cm² distance between their centres is
6cm. Find the radii of circles.
3. A pendulum swings through an angle of 300 and describes an arc 8.8cm in length. Find length of
pendulum.
4. The side of a square is 10cm find the area of circumscribed and inscribed the circle.
5. An Umbrella has 8 ribs which are equally spaced. Assume Umbrella to be flat circle of radius 45cm find the
area between two consecutive ribs of umbrella.(use π =3.14)
6. A child prepare a poster on “save energy” on a square sheet whose each side measure 60cm. at each corner
of the sheet, she draw a quadrant of radius 17.5cm in which she shows the ways to save energy at the
centre. She draws a circle of diameter 21cm and writes a slogan in it. Find the area of remaining
sheet.(use π =3.14)
7. A birthday cake is circular in shape. This cake is equally divided among six friends where radius of the cake is
60cm.(use π =3.14)
FInd the area of each piece of cake.

. ANSWER
LEVEL-I

1. (ii). 14/11
2. (ii). 128 cm2
3. (ii) 3π cm
4. (iv) 300
5. (iv) 1500
6. (ii)π/4
7. 1000
8. 36 cm
9. 3000
10. 154/3 cm2
LEVEL- II
2
1. 145.33 cm
2. D= 160 m
3. 4.71 cm2
4. 154 cm2
5. 44 cm
LEVEL- III
1. (i) 77 m2
(ii) 259 m2
2. 109.7 cm2
3. 98 cm2
4. 800
5. 7.77 cm2
SELFEVALUATION
1. 11 cm and 3
cm
2. 4 cm and 10
cm
3. 16.8 cm
4. 50 π cm2, 25 π cm2
5. 794.81 cm2

6. Area of Remaining sheet=2292.19cm2


7.Area of each piece=1884cm2
SURFACE AREA AND VOLUMES

KEY CONCEPTS

1. CUBOID:
(I) TOTAL SURFACE AREA OF A CUBOID:2(LB+ BH +HL)
(II) Volume of a cuboid=L x B x H sq .units
(III) Diagonal of cuboid= b2h2 units
CUBE :
(IV) Total Surface Area of a Cube = 6a2 sq. units
(V) Volume of the Cube=a3 cubic units
(VI) Diagonal of cube is= a units
2. Right Circular Cylinder:
(I) Curved Surface Area=2πrh
(II) Total Surface Area=2 πr ( h +r )
(III) Volume = πr2h

3. Right Circular Hollow Cylinder:


(I) Area of each end= πRr2) [R and r be the external radius and internal radius]
(II) Curved Surface Area of Hollow Cylinder=2 πh(R+r)
(III) Total Surface Area= π(R + r) (2h +R - r)
(IV) Volume of material= πh( R2-r2 )

4. Sphere:
(I) Surface Area =4 πr2
4
(II) Volume= πr³
3
5. Hemisphere:
(I) Curved Surface Area=2 πr2
(II) Total Surface Area=3πr2
2
(III) Volume= πr3
3

6. Right Circular Cone:


(I) Curved Surface Area = πrl [l=Slant Height]
(II) Total Surface Area= πr ( l + r )sq units
2
(III) Volume =1 πr h
3
LEVEL WISE QUESTIONS

LEVEL-I
1. The Surface Area of a Sphere is 616 cm2. Find its radius.
2. A cylinder and a cone are of the same base radius and of the same height. Find the ratio of the curved surface area
of the cylinder to that of cone.
3. Two cones have their heights in the ratio1:3 and radii 3:1. What is the ratio of their volumes?
4. The radii of two cones are in the ratio 2:1 and their volumes are equal. What is the ratio their heights?
5. The diameter of a sphere is 6cm. it is melted and drawn into a wire of diameter 2mm. Find the length of the wire.
6. Find the curved surface area of a right circular cone of height 15 cm and base diameter is16cm.
7. Find the maximum volume of a cone that can be out of a solid hemisphere of radius r.

LEVEL-II
1. Metallic sphere of radii 6cm, 8cm and10cm respectively, are melted to form a single solid sphere.
Find the radius of the resulting sphere.
2. A 20m deep well with diameter 7m is dug and the earth from digging evenly spread out to form a
platform 22m by 14m. Find the height of the platform.
3. Two cubes of volume 64cm3 are joined end to end. Find the volume of the sphere.
4. The largest sphere is curved out of a cube of a side 7cm. Find the volume of the sphere.
5. A circus tent is cylindrical up to a height of 3m and conical above it. If the diameter of the base
is 105 m and the slant height of the conical part is 53m. Find the total canvas used in making
the tent.
6. A vessel is in the form of a hemispherical bowl mounted by a hollow cylinder. The diameter of the
sphere is 14cm and the total height of the vessel is 13cm.Find its capacity?
7. A solid toy is in the form of a right circular cylinder with a hemispherical shape at one end and a
cone at the other end. Their common diameter is 4.2cm and the height of the cylindrical and
conical position is 12cm and 7cm respectively. Find the volume of the solid toy.
8. A solid is in the shape of a cone standing on a hemisphere with both their radii being equal
to 1cm and the height of the cone is equal to its radius. Find the volume of the solid in terms
of n.

Level-III
1. A hemispherical depression is cut from one face of the cubical wooden block such that the diameter l of the
hemisphere is equal to the edge of the cube. Determine the surface area of the remaining solid.
2. A juice seller was serving his customers using glasses. The inner diameter of the cylindrical glass was 5cm, but
the bottom of the glass had a hemispherical raised portion which reduced the capacity of the glass. If the
height of glass was 10cm, find what the apparent capacity of the glass was and what the actual capacity
was.(use n = 3.14)
3. The height of a cone is 30cm. A small cone is cut off at the top by a plane parallel to the base of its
volume be 1/27 of the volume of the given cone, at what height above the base is the section made?
4. A solid wooden toy is in the shape of a right circular cone mounted on a hemisphere. If the radius of the
hemisphere is 4.2cm and the total height of the toy is 10.2cm. Find the volume of the wooden toy.

SELF-EVALUATION
1. A tent is of the shape of a right circular cylinder up to a height of 3m and then becomes a right
circular cone with a maximum height of 13.5m, above the ground. Calculate the cost of painting
the inner side of the tent at the rate of Rs.2 per sq. metre, if the radius of the edge is14 metres.

2. If the radii of the circular ends of a conical bucket which is 45cm high are 28cm and 7cm. Find
the capacity of the bucket.

3. A pen stand made of wood is in the shape of a cuboid with four conical depression‟s to hold pens.
The dimensions of the cuboid are 15cm by 10cm by 3.5cm. The diameter of each of the
depression is 1cm and the depth is 1.4cm. Find the volume of the wood in the entire stand.
4. Three cubes each of side 5cm are joined end to end. Find the surface area of the resulting cuboid.

5. The diameter of a metallic sphere is 6cm. The sphere is melted and drawn into a wire of
uniform cross- section. If the length of the wire is 36m. Find its radius.

6. If the diameter of cross-section of a wire is decreased by 5%. How much percent will the length
be increased so that the volume remains the same?
ANSWER
LEVEL-I

1. 7 cm
2. √2:1
3. 3:1
4. 1:4
5. 3600cm or 36m
6. 427.04 cm²
7. 1/3 πr³

LEVEL- II
1. 12cm
2. 2.5 m
3. 128cm³
4. 179.67cm³
5. 9735m²
6. 1642.67cm³
7. 218.064cm³
8. n

LEVEL- III
1.l² (24+n)
2. 163.54cm³
3. 20 cm
4. 266.12cm³
SELF EVALUATION
1. Total area 1034m²,
Cost of painting Rs.2068

2. 48510cm3
3. 523.53 cm3
4. 350 cm2
5. 1mm
6. 10.8%
STATISTICS

(i) Assumed Mean method or Short cut method


∑n f i di
Mean= =a+ Where a = assumed mean And d = X - a
i=1 i i
n
i=1

 Median of a grouped frequency distribution can be calculated by Median

= l+ xh

Where
l=lower limit of median class
n= number of observations
cf=cumulative frequency of class preceding the median class
f=frequency of median class
h=class size of the median class.

 Mode of grouped data can be calculated by the following formula.

f1–f0
Mode=l+ ( )×h
2f1 –f0 –f2
Where
l=lower limit of modal class
h=size of class interval
f1 = Frequency of the modal class
fo =frequency of class preceding the modal class
f2=frequency of class succeeding the modal class

 Empirical relationship between the three measures of central tendency.


 3Median=Mode+2Mean
Or, Mode = 3 Median – 2 Mean
LEVEL – I

S. No Questions
1 What is the mean of first ten prime numbers?
2 A teacher asks the students to find the average marks obtained by the students in the class. What
the students will calculate : mean , mode and median ?
3 If the mode of a data is 45 and mean is 27, then median is .
4 Find the mode of the following
Xi 35 38 40 42 44
fi 5 9 10 7 2
5 Write the median class of the following distribution.
Class 0-10 10-20 20-30 30-40 40-50 50-60 60-70
Frequency 4 4 8 10 12 8 4
6 The wickets taken by a bowler in10 cricket matches are as follows:2,6,4,5,0,2,1,3,2,3 .Find the
mode of the data
7. What measure of central tendency is represented by the middle most observation of the data ?
8. What important information one can get by the abscissa of the point of intersection of the less than type and the
more than type cumulative frequency curve of a group data

LEVEL – II

S.No Questions Ans


1 Find the median of the following frequency distribution 167
Height in cm 160-162 163-165 166-168 169-171 172-174
Frequency 15 117 136 118 14

2. Given below is the distribution of IQ of the 100 students .Find the median of IQ 106.1
IQ 75-84 85-94 95-104 105-114 115-124 125-134 135-144
11 26 31 18

3 Find the median of the following distribution 28.5


Class 0-10 10-20 20-30 30-40 40-50 50-60
interval
frequency 20 15

4 A class teacher has the following absentee records of 40 students of a class for the whole term
No of 0-6 6-10 10-14 14-20 20-28 28-38 38-40
days
No of 11 10

Write the above distribution as less than type cumulative frequency distribution
5 Using the assumed mean method find the mean of the following data 27.2
0-10 10-20 20-30 30-40 40-50
frequency 12 13 10

6 Name the key terms used in central tendency Mean


Median
mode

LEVEL – III
SN Question Ans

Class 0-10 10-20 20-30 30-40 40-50

Frequency 5 18 15 P 6

Class 0-10 10-20 20-30 30-40 40-50

7 12 13 10 8
Frequency

Class 20-30 30-40 40-50 50-60 60-70 70-80 80-90


Frequency 25 15 P 6 24 12 8

Marks Less Less than Less than Less Less than Less than Less than Less than
than 10 30 50 than70 90 110 130 150

Frequency 0 10 25 43 65 87 96 100

A=18.93
23 B=18.83
Group A 50 78 46 28
Group B 54 89 40 25 17
SELF – EVALUATION

1. If mean=60 and median=50, then find mode using empirical relationship.


2. Find the value of p, if the mean of the following distribution is 18.
Variate (xi) 13 15 17 19 20+p 23
Frequency (fi) 8 2 3 4 5p 6

3. Find the mean, mode and median for the following data.
Classes 0-10 10-20 20-30 30-40 40-50 50-60 60-70
frequency 5 8 15 20 14 8 5
4. The median of the following data is 52.5. Find the value of x and y, if the total frequency is100.

Class Interval 0-10 10-20 20-30 30-40 40-50 50-60 60-70 70-80 80-90 90-100

frequency 2 5 X 12 17 20 Y 9 7 4

5. Find the mean marks for the following data.

Marks Below Below Below Below Below Below Below Below Below Below
10 20 30 40 50 60 70 80 90 100
No. of 5 9 17 29 45 60 70 78 83 85
students

6. The following table shows age distribution of persons in a particular region. Calculate the median age.

Age in Below Below Below Below Below Below Below Below


years 10 20 30 40 50 60 70 80
No. of 200 500 900 1200 1400 1500 1550 1560
persons

7. If the median of the following data is 32.5. Find the value of x and y.

Class 0-10 10-20 20-30 30-40 40-50 50-60 60-70 Total


Interval
frequency x 5 9 12 y 3 2 40
Value Based Question

Q1.The following frequency distribution gives the monthly consumption of electricity of 68 consumers of alocality.
Monthly 65 – 85 85 – 105 105 – 125 125- 145 145- 165 165 – 185 185 – 205
consumption (in
units)

Number of 4 5 13 20 14 8 4
consumers

Mr.Sharma always saves electricity by switching off all the electrical equipments just immediately after their uses. So,
his family belongs to the group 65-85.
(i) Find the median of the above data
(ii) How many families consumed 125 or more units of electricity during a month?
(iii) What moral values of Mr.Sharma have been depicted in this situation?

Q2.The mileage (km per litre) of 50 cars of the same models is tested by manufacturers and details are tabulated as
given below:-

Mileage (km per


10 – 12 12 – 14 14 -16 16- 18
litre)

12
No. of cars 7 18 13

i. Find the mean mileage.

ii. The manufacturer claims that the mileage of the model is16 km/litre. Do you agree with this claim?

iii. Which values do you think them manufacturer should imbibe in his life?
ANSWER
1. 12.9

2. MEAN

3. 33

4. MODE =40

5. MEDIAN=30-40

6. 2

7. Median

8. Median
Level II
Q1 -167

Q2 - 106.1

Q.3 - 28.5l

Q4

No. of days Less Less Less Less Less Less Less


Than 6 Than 10 Than 14 Than 20 Than 28 Than 38 Than 40

No. of 11 21 28 32 36 39 40
students

Q5- 27.2

Q6- Mean, Median, Mode


PROBABILITY
KEY POINTS
1. Probability:-The theoretical probability of an event E, written as P(E) is defined as.
P(E)=Number of out comes Favourable to E
Number of all possible out comes of the experiment
When we assume that the out comes of the experiment are equally likely.

2. The probability of a sure event (or certain event) is 1.


3. The probability of an impossible event is 0.
4. The probability of an Event E is number P(E) such that 0≤P(E)≤1.
5. Elementary events:- An event having only one outcome is called an elementary event. The sum of the
probabilities of all the elementary events of an experiment is 1.
6. For any event E, P(E)+P( ) = 1, where stands for not E, E and are called complementary event.
7. Performing experiments:-
a. Tossing a coin.
b. Throwing a die.
c. Drawing a card from deck of 52 cards.
8. Sample space:-The set of all possible outcomes in an experiment is called sample space.
9. An event is a subset of a sample space.
10. Equally likely events- If one event cannot be expected in preference to other event then they are said to be
Equally likely.

LEVEL-I
1. The probability of getting bad egg in a lot of 400 is 0.035. Then find the number of bad eggs in the lot.
2. Write the probability of a sure event.
3. What is the probability of an impossible event?
4. When a dice is thrown, and then find the probability of getting an odd number less than 3.
5. A girl calculates that the probability of her winning the third prize in a lottery is 0.08. If 6000 tickets are sold, how
many ticket has she bought.
6. What is probability that a non-leap year selected at random will contain 53 Sundays.
7. A jar contains 54 marbles each of which is blue ,green or white . The probability of selecting a blue marble at

random from the jar is 1/3, and the probability of selecting a green marble at random is 4/9 . How many white

marbles does the jar contain.

8. two coins are tossed simultaneously. Find the probability of getting exactly one head.
9. A card is drawn from a well shuffled deck of 52cards. Find the probability of getting an ace.
10. In a lottery, there are10 prizes and 25 blanks. Find the probability of getting a prize.
LEVEL-II
1. Find the probability of prime numbers selected at random from the numbers 3,4,5,6...25 .
2. A bag contains 5 red, 4 blue and 3 green balls. A ball is taken out from the bag at random. Find the probability that
the selected ball is (a) of red colour (b) not of green colour.
3. A card is drawn at random from a well-shuffled deck of playing cards. Find the probability of drawing
(a) A face card(b)card which is neither a king nor a red card
4. A dice is thrown once. What is the probability of getting a number greater than4?
5. Two dice are thrown at the same time. Find the probability that the sum of two numbers appearing on the top of the
dice is more than 9.
6. Two dice are thrown at the same time. Find the probability of getting different numbers on both dice.
7. A coin is tossed two times. Find the probability of getting almost one head.
8. Cards with numbers 2 to 101 are placed in a box. A card selected at random from the box. Find the probability that the
card which is selected has a number which is a perfect square.
9. Find the probability of getting the letter M in the word “MATHEMATICS”.

LEVEL-III
1. Cards bearing numbers 3,5,7…35 are kept in a bag. A card is drawn at random from the bag. Find the probability of
getting a card bearing (a) a prime number less than 15(b) a number divisible by 3 and 5.
2. Two dice are thrown at the same time. Find the probability of getting (a) same no. on the both side(b) different no. on
both dices.
3. A child game has 8 triangles of which three are blue and rest are red and ten squares of which six are blue and rest
are red. One piece is drawn at random. Find the probability of that is (a) A square (b) A triangle of red colour.
4. Two dice are thrown simultaneously. What is the probability that:
(a) 5 will not come up either of them? (b) 5 will come up on at least one? (c) 5 will come at both dice?
5. The king, queen and jack of clubs are removed from a deck of 52 playing cards and remaining cards are shuffled. A
card is drawn from the remaining cards. Find the probability of getting a card of (a) heart(b) queen(c) clubs
6. A game consists of tossing a one-rupee coin 3 times and noting its outcome each time. Hanif wins if all the tosses ,give
the same result,i.e.,3 heads or three tails and loses otherwise. Calculate the probability that Hanif will lose the game.
7. Cards bearing numbers 1,3,5…37 are kept in a bag. A card is drawn at random from the bag. Find the
Probability of getting a card bearing

(a) A prime number less than15


(b) A number divisible by 3 and 5.

8. A dice has its six faces marked 0,1,1,1,6,6. Two such dice are thrown together and total score is
recorded.(a) how many different scores are possible? (b) What is the probability of getting a total of seven?

Self-Evaluation/HOTS

1. Two dice are thrown simultaneously. Find the probability of getting an even number as the sum.
2. Cards marked with the number 2 to 101 are placed in a box and mixed thoroughly. One card is drawn from the
box. Find the probability that the number on the card is:
(i) An even number
(ii) A number less than14
(iii) A number is perfect square
(iv) A prime number less than 20
3. Find the probability that a leap year selected at random will contain 53 Sundays.
Value based Question
Q1.In a survey, it was found that 40% people use petrol, 35% uses diesel and remaining uses CNG for their vehicles.
Find the probability that a person uses CNG at random.
(a) Which fuel out of above 3 is appropriate for the welfare of the society?

Level -I
1. A die is thrown once. What is probability of getting a number greater than4?
2. A bag contains 4 red and 6 blackballs. A ball is taken out of the bag at random. Find the probability of getting a
blackball?
3. A die is thrown once. Find the probability of getting.
a) Prime number
b) A number divisible by 2.

Level -II
1. A bag contains card which are numbered from 2 to 90. A card is drawn at random from the bag. Find
the probability that it bears.

a.) A Two digit number


b.) A number which is perfect square.
2. Two dice are rolled once. Find the probability of getting such numbers on the two dice whose product is12.
Level – III
1. Red queens and black jacks are removed from a pack of 52 playing cards. A card is drawn at random from the
remaining card, after reshuffling them. Find the probability that the drawn card is:
(i) King ii) of red colour iii) a face card iv) queen
2. All the red face cards are removed from a pack of 52 playing cards. A card is drawn at random from the
remaining cards after reshuffling them. Find the probability that the card drawn is
(i) Of red colour ii) a queen iii) an ace iv) a face card.
3. In a family of 3 children, find the probability of having at least 1 boy.
4. Three unbiased coins are thrown simultaneously. Find the probability of getting.
i. Exactly two heads.
ii. At least two heads.
iii. At most two heads.
ANSWER
LEVEL-I
1. 14
2. 1
3. 0
4. 1/6
5. 480
6. 1/7
7. 12
8. ½
9. 1/13
10. 2/7
LEVEL - II
1. 8/23
2. A. 5/12 B. ¾
3. A. 3/13 B. 6/13
4. 1/3
5. 1/6
6. 5/6
7. ¾
8. 9/100
9. 2/11
LEVEL– III
1. A.5/17 B.1/17
2. A. 1/6 B. 5/6
3. A. 5/9 B. 5/18
4. A. 25/36 B. 11/36 C. 1/36
5. A. 13/49 B. 3/49, C 10/49
6. ¾
7. A.5/19 B.1/19
8. A. 6 scores B. 1/3
SELF EVALUATION

1. ½
2. A. ½ B 3/25 C 9/100, D. 2/25
3. 2 / 7

VALUE BASED QUESTION


1. Probability =0.25
CNG

BOARD QUESTION
LEVEL-I

1.1/3
2.3/5
3. ½, ½

LEVEL- II

1. 81/89, 8/89
2. 1/9
LEVEL- III

1. 1/12, 24/48, 1/6, 1/24


2. 10/23, 1/23,2/23,3/23
3. 7/8

4. 3/8, ½, 7/8

*****************

103
ACTIVITIES
Activity1: To find the HCF of two Numbers .

Activity2: To Draw the Graph of a Quadratic Polynomial and observe:


i. The shape of the curve when the coefficient of x2 is positive
ii. The shape of the curve when the coefficient of x2 is negative
iii. Its number of zero

Activity3: To obtain the zero of a linear Polynomial Geometrically


Activity4: To obtain the condition for consistency of system of linear Equations in two variables Activity5:

To find geometrically the solution of a Quadratic Equation ax2+bx++c=0, a 0


(where a=1) by using the method of computing the square.
Activity6: To verify that given sequence is an A .P(Arithmetic Progression) by the paper
Cutting and Paper Folding.

Activity7: To verify by Graphical method

Activity8: To Draw a System of Similar Squares, Using two intersecting Strips with nails
Activity9: To Draw a System of similar Triangles Using Y shaped Strips with nails
Activity10: To verify Basic proportionality theorem using parallel line board
Activity11: To verify the theorem Ratio of the Areas of Two Similar Triangles is Equal to the
Ratio of the Squares of their corresponding sides through paper cutting.

Activity12: To verify Pythagoras Theorem by paper cutting, paper folding and adjusting (Arranging)
Activity13: Verify that two figures (objects)having the same shape (and not necessarily the same
size) are similar figures. Extend the similarity criterion to Triangles.
Activity14: To find the distance between two objects by physically demonstrating the position
of the two objects say two Boys in a Hall, taking a set of reference axes with the
corner of the hall as origin.
Activity15: Division of line segment by taking suitable points that intersects the axes at some Points
and then verifying section formula.
Activity16: To verify the formula for the area of a triangle by graphical method.
Activity17: (a ) To make mathematical instrument clinometers (or sextant )for
measuring the angle of elevation/depression of an object
Activity 18: To calculate the height of an object making use of clinometers (or sextant)

Activity19: To verify experimentally that the tangent at any point to a circle is


perpendicular to the Radius through that point.
104
Activity20: To find the number of tangent from a point to the circle
Activity21: To verify that lengths of tangents drawn from an external Point to a circle are
equal by using method of paper cutting, paper folding and pasting.
Activity22: To Draw a quadrilateral similar to a given quadrilateral as per given scale factor (Less than1)

Activity23: To obtain formula for Area of a circle experimentally.


Activity24: To give a suggestive demonstration of the formula for the surface Area of a circus Tent.
Activity25: To obtain the formula for the volume of Frustum of a cone.
Activity26: To find the Average Height (in cm) of students studying in a school.
Activity27: To Draw a cumulative frequency curve (or an ogive) of less than type.
Activity28: To Draw accumulative frequency curve (or an ogive) of more than type.

Activity29: To get familiar with the idea of probability of an event through a double color card experiment.
Activity30: To verify experimentally that the probability of getting two tails when two coins are tossed
simultaneously is ¼ = (0.25) By eighty tosses of two coins.

105 | P a g e
ORAL TEST (REAL NUMBERS)

Answer the following questions:


1. Euclid‟s division algorithm is a technique to compute the _of two given
positive integers.
2. HCF (124, 24) is .
3. “Every composite number can be expressed (factorised) as a product of primes, and this
factorization is unique, apart from the order in which the prime factors occurs”. The above
statement is called .
4. For any two positive integers a and b,
a x b=HCF(a, b) x
5. If a number cannot be written in the form p/q, where p and q are integers and q ≠0, then
it is called _.

QUIZ
(POLYNOMIALS)
Answer the following questions:
1. What is a quadratic polynomial?
2. What is the degree of a quadratic polynomial?
3. What are the zeroes of a polynomial?
4. What is the shape of curve of a quadratic polynomial graph?
5. State remainder theorem.

ORAL TEST
1. If P(x) is a polynomial in x, the highest power of x in P(x) is called the _of the
polynomial P(x).
2. A polynomial of degree 2 is called a .
3. The linear polynomial ax + b, a≠0, has exactly one zero, namely, the x-coordinate of the
point where the graph of y= ax+ b intersects the _.
4. A polynomial P(x) of degree n has atmost _zeroes.
5. The sum and the product of the zeroes of a quadratic polynomial x2+7x+10 is _
and _.

QUIZ
(Pair of linear equations in two variables)
Answer the following questions:
1. What is a pair of line of equations in two variables?
2. Give the general form of a pair of linear equation?
3. What are the methods of solving a pair of linear equation in two variables?
4. What is the condition for inconsistent solution?
5. What is the shape of curve in graph of a linear equation?
Oral Test

1. Every solution(x, y) of a linear equation in two variables, ax + by + c = 0 corresponds to a


on the line representing the equation, and vice versa.
2. If the pair of linear equations in two variables have only one common point on both the
lines, then we have a solution.
3. A pair of equations which has no solution is called a/an _pair of linear
equations.
4. Half the perimeter of a rectangular garden, whose length is 4m more than its width is
36m. The dimension of the garden are and _.
5. A pair of linear equations in two variables can be represented and solved by the
graphical method and _method.

QUIZ
(Triangles)

1. What is SAS similarity criterion?


2. What is the relationship between congruency and similarity of figures?
3. What are the criteria for the similarity of two triangles?
4. For what types of triangles is Pythagoras theorem applicable?
5. What is the name of Basic Proportionality Theorem?

ORAL TEST

1. All triangles are similar (equilateral / isosceles / Scalene)


2. The longest side of a right angled triangle is called _.
3. In a _______ square of the hypotenuse is equal to the sum of squares of the other
two sides.
4. In the given figure, if DE || BC, then the value of x is _

107
5. State whether the following quadrilateral are similar or not.

QUIZ
(Introduction to Trigonometry)

1. What is trigonometry?
2. What are trigonometric ratios of an acute angle in a right triangle?
3. From the figure find the value of cos A.

4. Write the trigonometric ratios of 60o.


5. Evaluate: 3 sin230o + 2 tan260o – 5 cos2 60o.

ORAL TEST

1. In a right triangle ABC, right angles at B , sin A = _.


2. Tan2A = , for 0o ≤ A < 90o.
3. Sec2A- = 1, for 0o ≤ A < 90o.
4. If cot A= 7/8, then (1+ sin ) (1 – sin )/(1 + cos )(1 – cos )
5. (1–tan245o) / (1+tan245o) =

108
QUIZ
(STATIST
ICS)
1. Name the measures of central tendency.
2. What is cumulative frequency?
3. How will you represent the cumulative frequency distribution graphically?
4. How will you find the median of a grouped data ?
5. What is the empirical relationship between the three measures of
central tendency.

ORAL TEST
1. _ _is the sum of the values of all the observations divided by
the total number of observations.
2. Class mark = _/2.
3. The formula for finding the mean using the assumed mean method is .
4. The formula for finding the mode in a grouped frequency distribution is .
5. The formula for finding the median of grouped data is _.

Page 1 of 14
Class- X Session- 2020-21
Subject- Mathematics -Standard
Sample Question Paper
Time Allowed: 3 Hours Maximum Marks: 80

General Instructions:
1. This question paper contains two parts A and B.
2. Both Part A and Part B have internal choices.

Part – A:
1. It consists three sections- I and II.
2. Section I has 16 questions of 1 mark each. Internal choice is provided in 5 questions.
3. Section II has 4 questions on case study. Each case study has 5 case-based sub-parts. An
examinee is to attempt any 4 out of 5 sub-parts.
Part – B:
1. Question No 21 to 26 are Very short answer Type questions of 2 mark each,
2. Question No 27 to 33 are Short Answer Type questions of 3 marks each
3. Question No 34 to 36 are Long Answer Type questions of 5 marks each.
4. Internal choice is provided in 2 questions of 2 marks, 2 questions of 3 marks and 1 question of 5
marks.
Question Part-A Marks
No. allocated
Section-I

Section I has 16 questions of 1 mark each. Internal choice is provided


in 5 questions.

1 If xy=180 and HCF(x,y)=3, then find the LCM(x,y). 1

OR

14587
The decimal representation of will terminate after how many decimal
21 × 5 4
places?

2 If the sum of the zeroes of the quadratic polynomial 3x2-kx+6 is 3, then find 1
the value of k.

Page 2 of 14
3. For what value of k, the pair of linear equations 3x+y=3 and 6x+ky=8 does 1
not have a solution.

4. If 3 chairs and 1 table costs Rs. 1500 and 6 chairs and 1 table costs Rs.2400. Form 1
linear equations to represent this situation.

5. Which term of the A.P. 27, 24, 21,…..is zero? 1

OR

In an Arithmetic Progression, if d= - 4, n=7,an=4, then find a.

6. For what values of k, the equation 9x2+6kx+4=0 has equal roots?

7. Find the roots of the equation x2+7x+10=0 1

OR

For what value(s) of „a‟ quadratic equation 30 ��2 − 6� + 1 = 0 has no real


roots?

8. If PQ=28cm, then find the perimeter of ∆PLM 1

9. If two tangents are inclined at 60˚ are drawn to a circle of radius 3cm then 1
find length of each tangent.

OR

PQ is a tangent to a circle with centre O at point P. If ∆OPQ is an isosceles


triangle, then find ∠ OQP.

10. In the ∆ABC, D and E are points on side AB and AC respectively such that 1
DE II BC. If AE=2cm, AD=3cm and BD=4.5cm, then find CE.

Page 3 of 14
11. In the figure, if B1, B2, B3,…... and A1,A2, A3,….. have been marked at 1
equal distances. In what ratio C divides AB?

12. Sin A + Cos B = 1, 𝐴 = 30° and B is an acute angle, then find the value of B. 1

13. If x=2sin2Ɵ and y=2cos2Ɵ+1, then find x+y 1

14. In a circle of diameter 42cm,if an arc subtends an angle of 60˚ at the centre 1
where ∏=22/7, then what will be the length of arc.

15. 12 solid spheres of the same radii are made by melting a solid metallic 1
cylinder of base diameter 2cm and height 16cm. Find the diameter of the
each sphere.

16. Find the probability of getting a doublet in a throw of a pair of dice. 1

OR

Page 4 of 14
Find the probability of getting a black queen when a card is drawn at random
from a well-shuffled pack of 52 cards.

Section-II

Case study based questions are compulsory. Attempt any four sub
parts of each question. Each subpart carries 1 mark

17. Case Study based-1


SUN ROOM

The diagrams show the plans for a sun room. It will be built onto the wall of a
house. The four walls of the sunroom are square clear glass panels. The roof
is made using
 Four clear glass panels, trapezium in shape, all the same size
 One tinted glass panel, half a regular octagon in shape

(a) Refer to Top View 1


Find the mid-point of the segment joining the points J (6, 17) and I (9, 16).
(i) (33/2,15/2)
(ii) (3/2,1/2)
(iii)(15/2,33/2)
(iv) (1/2,3/2)

Page 5 of 14
(b) Refer to Top View 1
The distance of the point P from the y-axis is
(i) 4
(ii) 15
(iii) 19
(iv) 25

(c) Refer to Front View 1


The distance between the points A and S is
(i) 4
(ii) 8
(iii)16
(iv)20
(d) Refer to Front View 1
Find the co-ordinates of the point which divides the line segment joining the
points A and B in the ratio 1:3 internally.
(i) (8.5,2.0)
(ii) (2.0,9.5)
(iii) (3.0,7.5)
(iv) (2.0,8.5)

(e) Refer to Front View 1


If a point (x,y) is equidistant from the Q(9,8) and S(17,8),then
(i) x+y=13
(ii) x-13=0
(iii) y-13=0
(iv)x-y=13

18. Case Study Based- 2


SCALE FACTOR AND SIMILARITY
SCALE FACTOR
A scale drawing of an object is the same shape as the object but a different
size.
The scale of a drawing is a comparison of the length used on a drawing to
the length it represents. The scale is written as a ratio.
SIMILAR FIGURES
The ratio of two corresponding sides in similar figures is called the scale
factor.
Length in image
Scale factor =
Corresponding length in object
If one shape can become another using Resizing then the

shapes are Similar


Th

Page 6 of 14
Th

Rotation or Turn

Reflection or Flip

Translation or Slide

Hence, two shapes are Similar when one can become the other after
a resize, flip, slide or turn.

(a) A model of a boat is made on the scale of 1:4. The model is 120cm long. The 1
full size of the boat has a width of 60cm. What is the width of the scale
model?

(i) 20 cm
(ii) 25 cm
(iii) 15 cm
(iv)240 cm

Page 7 of 14
(b) What will effect the similarity of any two polygons? 1
(i) They are flipped horizontally
(ii)They are dilated by a scale factor
(iii)They are translated down
(iv)They are not the mirror image of one another

(c) If two similar triangles have a scale factor of a: b. Which statement regarding 1
the two triangles is true?
(i)The ratio of their perimeters is 3a : b
(ii)Their altitudes have a ratio a:b
a
(iii) Their medians have a ratio : b
2
2 2
(iv) Their angle bisectors have a ratio a : b
(d) The shadow of a stick 5m long is 2m. At the same time the shadow of a tree 1
12.5m high is

(i)3m
(ii)3.5m
(iii)4.5m
(iv)5m

(e) Below you see a student's mathematical model of a farmhouse roof with 1
measurements. The attic floor, ABCD in the model, is a square. The beams
that support the roof are the edges of a rectangular prism, EFGHKLMN. E is
the middle of AT, F is the middle of BT, G is the middle of CT, and H is the
middle of DT. All the edges of the pyramid in the model have length of 12 m.

Page 8 of 14
What is the length of EF, where EF is one of the horizontal edges of
the block?
(i) 24m
(ii) 3m
(iii) 6m
(iv)10m
19. Case Study Based- 3
Applications of Parabolas-Highway
Overpasses/Underpasses A highway underpass is parabolic
in shape.

Parabola
A parabola is the graph that
results from p(x)=ax2+bx+c
Parabolas are symmetric
about a vertical line known
as the Axis of Symmetry.
The Axis of Symmetry runs
through the maximum or
minimum point of the
parabola which is called the

Page 9 of 14
Vertex

(a) If the highway overpass is represented by x2–2x –8. Then its zeroes are
(i) (2,-4)
(ii) (4,-2)
(iii) (-2,-2)
(iv) (-4,-4)

(b) The highway overpass is represented graphically.


Zeroes of a polynomial can be expressed graphically. Number of zeroes of
polynomial is equal to number of points where the graph of polynomial
(i) Intersects x-axis
(ii) Intersects y-axis
(iii) Intersects y-axis or x-axis
(iv)None of the above

Page 10 of 14
(c) Graph of a quadratic polynomial is a
(i) straight line
(ii) circle
(iii)parabola
(iv)ellipse

(d) The representation of Highway Underpass whose one zero is 6 and sum of
the zeroes is 0, is
(i)x2 – 6x + 2
(ii) x2 – 36
(iii)x2 – 6
(iv)x2 – 3

(e) The number of zeroes that polynomial f(x) = (x – 2)2 + 4 can have is:
(i)1
(ii) 2
(iii) 0
(iv) 3

20. Case Study Based- 4


100m RACE
A stopwatch was used to
find the time that it took a
group of students to run 100
m.

Time 0-20 20-40 40-60 60-80 80-100


(in sec)
No. of 8 10 13 6 3
students

Page 11 of 14
(a) Estimate the mean time taken by a student to finish the race.

(i) 54
(ii) 63
(iii)43
(iv)50

(b) What wiil be the upper limit of the modal class ?

(i) 20
(ii) 40
(iii)60
(iv)80
(c) The construction of cummulative frequency table is useful in determining the

(i)Mean
(ii)Median
(iii)Mode
(iv)All of the above

(d) The sum of lower limits of median class and modal class is

(i)60
(ii)100
(iii)80
(iv)140

(e) How many students finished the race within 1 minute?

(i) 18
(ii) 37
(iii) 31
(iv)8

Part –B
All questions are compulsory. In case of internal choices, attempt any
one.

21. 3 bells ring at an interval of 4,7 and 14 minutes. All three bell rang at 6 am, 2
when the three balls will the ring together next?

22. Find the point on x-axis which is equidistant from the points (2,-2) and (-4,2) 2

OR

Page 12 of 14
P (-2, 5) and Q (3, 2) are two points. Find the co-ordinates of the point R on
PQ such that PR=2QR

23. Find a quadratic polynomial whose zeroes are 5-3√2 and 5+3√2. 2

24. Draw a line segment AB of length 9cm. With A and B as centres, draw 2
circles of radius 5cm and 3cm respectively. Construct tangents to each circle
from the centre of the other circle.

25. If tanA=3/4, find the value of 1/sinA+1/cosA 2

OR

If √3 sinƟ-cosƟ=0 and 0˚<Ɵ <90˚, find the value of Ɵ

26. In the figure, quadrilateral ABCD is circumscribing a circle with centre O 2


and AD⊥ AB. If radius of incircle is 10cm, then the value of x is

27.. Prove that 2-√3 is irrational, given that √3 is irrational. 3

28. If one root of the quadratic equation 3x2+px+4=0 is 2/3, then find the value 3
of p and the other root of the equation.

OR

The roots α and β of the quadratic equation x2-5x+3(k-1)=0 are such that α-
β=1. Find the value k.

Page 13 of 14
29. In the figure, ABCD is a square of side 14 cm. Semi-circles are drawn with 3
each side of square as diameter. Find the area of the shaded region.

30. The perimeters of two similar triangles are 25cm and 15cm respectively. If 3
one side of the first triangle is 9cm, find the length of the corresponding side
of the second triangle.

OR

In an equilateral triangle ABC, D is a point on side BC such that BD = 1/3


BC. Prove that 9 AD2 = 7 AB2

31. The median of the following data is 16. Find the missing frequencies a and b, 3
if the total of the frequencies is 70.

Class 0-5 5-10 10-15 15-20 20-25 25-30 30-35 35-40


Frequency 12 a 12 15 b 6 6 4

32. 3

If the angles of elevation of the top of the candle from two coins distant „a‟
cm and „b‟ cm (a>b) from its base and in the same straight line from it are
30˚ and 60˚, then find the height of the candle.

Page 14 of 14
Section V
33. The mode of the following data is 67. Find the missing frequency x. 3

Class 40-50 50-60 60-70 70-80 80-90


Frequency 5 x 15 12 7

34. The two palm trees are of equal heights and are standing opposite each 5
other on either side of the river, which is 80 m wide. From a point O
between them on the river the angles of elevation of the top of the trees
are 60° and 30°, respectively. Find the height of the trees and the
distances of the point O from the trees.

OR

The angles of depression of the top and bottom of a building 50 meters


high as observed from the top of a tower are 30˚ and 60˚ respectively.
Find the height of the tower, and also the horizontal distance between the
building and the tower.

35. Water is flowing through a cylindrical pipe of internal diameter 2cm, into a 5
cylindrical tank of base radius 40 cm at the rate of 0.7m/sec. By how
much will the water rise in the tank in half an hour?

36. A motorboat covers a distance of 16km upstream and 24km downstream 5


in 6 hours. In the same time it covers a distance of 12 km upstream and
36km downstream. Find the speed of the boat in still water and that of the
stream.

Page 1 of 10
MARKING SCHEME SQP
MATHEMATICS (STANDARD) 2020-21

CLASS X

S.NO. ANSWER MARKS


Part-A
1. (LCM)(3) =180 ½
LCM=60 ½

OR

Four decimal places 1

2. α+β=k/3 ½
3=k/3
K=9 ½

3. 3 1 3 ½
=≠
6 K 8
3 1
=
6 K
K=2 ½

4. Let the cost of 1 chair=Rs.x ½


And the cost of 1 table=Rs. y
3x+y=1500 ½
6x+y=2400

5. an=a+(n-1)d
0=27+(n-1)(-3) ½
30=3n
n =10 ½
10th

OR

an=a+(n-1)d
4=a+6x(-4) ½
a=-28 ½

6. 9x2+6kx+4=0
(6k)2-4X9X4=0 ½
36k2=144
K2=4
K=±2 ½

Page 2 of 10
7. x2+7x+10=0
x2+5x+2x+10=0 ½
(x+5)(x+2)=0
X=-5, x= - 2 ½

OR

3ax2-6x+1=0 ½
(-6)2-4(3a) (1)<0

12a>36 =>a>3 ½

8. PQ=PT
PL+LQ=PM+MT
PL+LN=PM+MN
Perimeter(∆PLM)
=PL+LM+PM ½
=PL+LN+MN+PM
=2(PL+LN)
=2(PL+LQ)
=2X28=56cm ½

9.

In ∆PAO ½
Tan30˚=AO/PA
1/√3 =3/PA ½
PA=3√3 cm

OR

In ∆OPQ
˂P+˂Q+˂O=180˚
2˂Q+˂P=180˚ ½
2˂Q+90˚=180˚
2˂Q=90˚
˂Q= 45˚ ½

Page 3 of 10
10. AD AE
=
BD CE
½
3 2
=
4.5 CE ½
CE=3cm

11. 8:5 1

12. Sin30˚+cosB=1
½+cosB=1 ½
CosB=1/2
B=60˚ ½

13. x+y
=2sin2Ɵ +2cos2Ɵ+1 ½
=2(sin2Ɵ +cos2Ɵ)+1
=3 ½

14. length of arc=Ɵ/360˚(2πr) ½


= 60/360(2X22/7X21)
=22 cm ½

15. πR2H=12X4/3πr3

1X1x16=4/3Xr3 X12 ½
r3=1
r=1
d=2cm ½

16. probability of getting a doublet=1/6 1

OR

probability of getting a black queen=2/52=1/26

17. (a) iii)(15/2,33/2) 1x4=4


(b) i) 4
(c) iii)16
(d) iv)(2.0,8.5)
(e) ii) x-13=0
18. (a) iii)15 cm 1x4=4
(b) iv)They are not the mirror image of one another
(c) ii)Their altitudes have a ratio a:b
(d) iv) 5m
(e) iii)6m
19. (a) ii) (4,-2) 1x4=4
(b) i) Intersects x-axis
(c) iii) parabola

Page 4 of 10
(d) ii) x2 – 36
(e) iii) 0
20. (a) iii)43 1x4=4
(b) iii)60
(c) ii)Median
(d) iii)80
(e) iii)31

Part-B
21. 4=2X2 ½
7=7X1
14=2X7 ½
½
LCM=2X2X7=28
½
The three bells will ring together again at 6:28 am
22. Let P(x,0) be a point on X-axis
PA=PB ½
PA2=PB2 ½
(x-2)2+(0+2)2=(x+4)2+(0-2)2
X2+4-4x+4=x2+16+8x+4
-4x+4=8x+16
X=-1 ½
P(-1,0) ½

OR

PR:QR=2:1 ½
1(−2)+2(3) 1(5)+2(2) 1
R( , )
2+1 2+1
R(4/3, 3) ½

23. Sum of zeroes= 5-3√2+5+3√2=10 ½


Product of zeroes= (5-3√2)(5+3√2)= 7 1
P(x)= X2-10x+7 ½

24. Line
seg=1/2

Circles=1
/2

Tangents
=1/2+
½

Page 5 of 10
25. tanA=3/4=3k/4k ½
sinA=3k/5k=3/5,cosA=4k/5k=4/5 ½
1/sinA+1/cosA
=5/3+5/4 ½
=(20+15)/12 ½
=35/12

OR

√3 sinƟ=cosƟ ½
sinƟ/cosƟ=1/√3 ½
tanƟ=1/√3 ½
Ɵ=30˚ ½

26. ˂A = ˂OPA =˂OSA = 90˚ ½


Hence, ˂SOP=90˚
Also, AP=AS
Hence, OSAP is a square
AP=AS=10cm ½
CR=CQ=27cm
BQ=BC-CQ=38-27=11cm ½
BP=BQ=11 cm
X=AB=AP+BP=10+11=21 cm ½

27. Let 2-√3 be a rational number ½


We can find co-prime a and b (b≠0) such that
2-√3=a/b ½
2-a/b=√3 ½
So we get,(2a-b)/b=√3
Since a and b are integers, we get (2a-b)/b is irrational and so
√3 is rational. But √3 is an irrational number ½
Which contradicts our statement ½
Therefore 2-√3 is irrational ½

28. 3x2+px+4=0 ½
3(2/3)2+p(2/3)+4=0
4/3+2p/3+4=0 ½
P=-8 ½
3x2-8x+4=0
3x2-6x-2x+4=0 ½
X=2/3 or x=2 ½
Hence, x=2 ½

Page 6 of 10
OR
α+β=5----- (1) ½
α-β=1 ----- (2) ½
Solving (1) and (2), we get
α=3 and β=2 ½
also αβ=6 ½
or 3(k-1)=6 ½
k-1=2
k=3 ½

29.
Area of 1 segment = area of sector –area of triangle ½
=( 90˚/360˚)πr2 – ½ x7x7
=1/4x22/7x72 – ½ x7x7 ½
= 14cm2 ½
Area of 8 segments=8x14= 112 cm2 ½
Area of the shaded region = 14x14-112 ½
=196-112=84cm2 ½
(each petal is divided into 2 segments)

30. ∆ABC~∆DEF
Perimeter (∆ABC)
=
AB+BC+CA
= AB 1
Perimeter (∆DEF) DE+EF+FD DE ½
25 9
=
15 X ½
X=5.4cm 1
DE=5.4cm

OR

Construction-Draw AM ḻ BC
½
BD ꓕ 1/3 BC , BM=1/2 BC
In ∆ABM,
AB2=AM2+BM2
½
=AM2+(BD+BM)2
=AM2+DM2+BD2+2BD. DM
½
=AD2+BD2+2BD(BM-BD)
=AD2+(BC/3)2+2. BC/3.(BC/2-BC/3)
=AD2+2BC2/9
½
=AD2+2AB2/9
Hence,7AB2=9AD2
½

Page 7 of 10
31. Class Frequency Cumulative 1
frequency
0-5 12 12
5-10 a 12+a
10-15 12 24+a
15-20 15 39+a
20-25 b 39+a+b
25-30 6 45+a+b
30-35 6 51+a+b
35-40 4 55+a+b
Total 70

55+a+b=70 ½
a+b=15
N
− cf
median=l+2 Xh ½
f
35−24−a
16 =15+ X5
15
1=(11-a)/3
A=8
½
½
55+a+b=70
55+8+b=70
B=7
32.
½

½
Let AB=candle
C and D are coins
Tan60˚=AB/BC=h/b
√3=h/b
H=b√3 ---------------- (1) ½
Tan30˚=AB/BD=h/a
1/√3=h/a
H=a/√3 --------------- (2) ½
Multiplying (1) and (2), we get
H2= b√3X a/√3 ½
H2= b a
H=√ab m ½

Page 8 of 10
33.
f1−f0
Mode= l+ xh ½
2f1−f2−f0 ½
67 = 60+ 15−x x 10
30−12−x ½
15−x
7= X 10
18−x ½
7x(18-x)=10(15-x)
126-7x=150-10x ½
3x=150-126
3x=24 ½
X=8
34. 1

Let BD=river ½
AB=CD=palm trees=h
BO=x ½
OD=80-x
In ∆ABO,
Tan60˚=h/x ½
√3=h/x ---------------------------------- (1)
H=√3x ½
In ∆CDO,
Tan 30˚=h/(80-x)
1/√3= h/(80-x) ------------------------ (2) ½
Solving (1) and (2), we get
X=20
H=√3x=34.6 ½
the height of the trees=h=34.6m
BO=x=20m ½
DO=80-x=80-20=60m
½

Page 9 of 10
OR

Let AB=Building of height 50m


RT= tower of height= h m ½
BT=AS=x m
AB=ST=50 m ½
RS=TR-TS=(h-50)m
In ∆ARS, tan30˚=RS/AS
1/√3 = (h-50)/x ------------------ (1)
In ∆RBT, tan60˚=RT/BT ½
√ 3 = h/x ----------------------------------(2)
½
Solving (1) and (2), we get ½
h= 75
from (2) ½
x=h/√3
=75/√3 ½
=25√3
Hence, height of the tower=h=75m
Distance between the building and the tower=25√3=43.25m ½

35. For pipe , r = 1cm ½


Length of water flowing in 1 sec, h=0.7m=7cm ½
Cylindrical Tank,R=40 cm , rise in water level=H ½
Volume of water flowing in 1 sec= πr2h=πx1x1x70
=70π ½
Volume of water flowing in 60 sec=70πx60
1
Volume of water flowing in 30 minutes=70πx60x30 ½

Volume of water in Tank=πr2H=πx40x40xH ½


½
Volume of water in Tank= Volume of water flowing in 30 ½
minutes
πx40x40xH = 70πx60x30
H=78.75cm

Page 10 of 10
36. Let speed of the boat in still water =x km/hr, and ½
Speed of the current =y km/hr
Downstream speed =(x+y) km/hr ½
Upstream speed =(x−y) km/hr ½
24 16 ½
+ = 6--------(1)
x+y x−y

36 12
+ = 6----------(2) ½
x+y x−y

1 1
Let = u and =v
x+y x−y

Put in the above equation we get,


24u+16v=6 ½
Or, 12u+8v=3 ... (3)
36u+12v=6
Or, 6u+2v=1 ... (4)
Multiplying (4) by 4, we get,
24u+8v=4v … (5)
Subtracting (3) by (5), we get, ½
12u=1
⇒u=1/12
Putting the value of u in (4), we get, v=1/4 ½
1 1 1 1
⇒ = and =
x+y 12 x-y 4
⇒x+y=12 and x−y=4
Thus, speed of the boat in still water = 8 km/hr, ½
Speed of the current = 4 km/hr ½

Page 1 of 27
Class- X
Mathematics-Basic (241)

Sample Question Paper 2020-


21
Max. Marks: 80 Duration:3 hours

General Instructions:

1. This question paper contains two parts A and B.


2. Both Part A and Part B have internal choices.

Part – A:
1. It consists of two sections- I and II
2. Section I has 16 questions. Internal choice is provided in 5 questions.
3. Section II has four case study-based questions. Each case study has 5 case-based sub-parts.
An examinee is to attempt any 4 out of 5 sub-parts.

Part – B:
1. Question No 21 to 26 are Very short answer Type questions of 2 mark each,
2. Question No 27 to 33 are Short Answer Type questions of 3 marks each
3. Question No 34 to 36 are Long Answer Type questions of 5 marks each.
4. Internal choice is provided in 2 questions of 2 marks, 2 questions of 3 marks and 1 question of 5
marks.

Questi Part-A Marks


on No.
Section-I

1
1. Express 156 as the product of primes.

1
2. Write a quadratic polynomial, sum of whose zeroes is 2 and product is -8.

1
3. Given that HCF (96,404) is 4, find the LCM ( 96,404).

OR

State the fundamental Theorem of Arithmetic.

Page 2 of 27
4 On comparing the ratios of the coefficients, find out whether the pair of 1
equations x – 2y =0 and 3x + 4y -20 =0 is consistent or inconsistent.

5 If a and b are co-prime numbers, then find the HCF (a, b). 1

6 1
Find the area of a sector of a circle with radius 6cm if angle of the sector is
60°. (Take π = 22/7)

OR

A horse tied to a pole with 28m long rope. Find the perimeter of the field where
the horse can graze. (Take π = 22/7)

7 In the given fig. DE || BC, ADE =70° and BAC=50°, then angle 1
BCA =

OR

In the given figure, AD = 2cm, BD = 3 cm, AE = 3.5 cm and AC = 7 cm. Is DE


parallel to BC ?

Page 3 of 27
8 The cost of fencing a circular field at the rate of Rs.24 per metre is Rs. 5280. 1
Find the radius of the field.

9 A tree breaks due to storm and the broken part bends so that the top of the 1
tree touches the ground where it makes an angle 30° . The distance between
the foot of the tree to the point where the top touches the ground is 8m. Find
the height of the tree from where it is broken.

10 If the perimeter and the area of a circle are numerically equal, then find the 1
radius of the circle

11 Write the empirical relationship among mean, median and mode. 1

12 To divide a line segment BC internally in the ratio 3 : 5, we draw a ray BX 1


such that ∠ CBX is an acute angle. What will be the minimum number of points
to be located at equal distances, on ray BX?

1
13 For what values of p does the pair of equations 4x + p y +8 =0 and 2x +2y +2
=0 has unique solution?

OR

What type of straight lines will be represented by the system of equations 2x +


3y =5 and 4x + 6y = 7 ?

14 A bag contains 3 red balls and 5 black balls. A ball is drawn at random from 1
the bag. What is the probability that the ball drawn is red?

OR

A die is thrown once. What is the probability of getting a prime number?

15 A tower stands vertically on the ground. From a point on the ground, which is 1
15m away from the foot of the tower, the angle of elevation of the top of the
tower is found to be 60°.Find the height of the tower.

16 Probability of an event E + Probability of the event E ( not E) is, 1

Page 4 of 27
Section-II
Case study-based questions are compulsory. Attempt any 4 sub parts
from each question. Each question carries 1 mark

17

Mathematics teacher of a school took her 10th standard students to show Red
fort. It was a part of their Educational trip. The teacher had interest in history
as well. She narrated the facts of Red fort to students. Then the teacher said
in this monument one can find combination of solid figures. There are 2 pillars
which are cylindrical in shape. Also 2 domes at the corners which are
hemispherical.7 smaller domes at the centre. Flag hoisting ceremony on
Independence Day takes place near these domes.

i) How much cloth material will be required to cover 2 big domes each of radius 1
2.5 metres? (Take π = 22/7)

a) 75m2 b) 78.57m2 c) 87.47m2 d) 25.8m2


b)
ii) Write the formula to find the volume of a cylindrical pillar. 1

a) πr2h b) πrl c) πr(l + r) d) 2πr

iii) Find the lateral surface area of two pillars if height of the pillar is 7m and 1
radius of the base is 1.4m.

a) 112.3cm2 b) 123.2m2 c) 90m2 d) 345.2cm2

iv) How much is the volume of a hemisphere if the radius of the base is 3.5m? 1

a) 85.9 m3 b) 80 m3 c) 98 m3 d) 89.83 m3

Page 5 of 27
v) What is the ratio of sum of volumes of two hemispheres of radius 1cm each to 1
the volume of a sphere of radius 2 cm?
a) 1:1 b) 1:8 c) 8 :1 d) 1:16

18 Class X students of a secondary school in Krishnagar have been allotted a


rectangular plot of a land for gardening activity. Saplings of Gulmohar are
planted on the boundary at a distance of 1m from each other. There is a
triangular grassy lawn in the plot as shown in the fig. The students are to sow
seeds of flowering plants on the remaining area of the plot.

Considering A as origin, answer question (i) to (v)

i) Considering A as the origin, what are the coordinates of A? 1

a) (0,1) b) (1,0) c) (0,0) d)(-1,-1)

ii) What are the coordinates of P? 1

a) (4,6) b)( 6,4) c) (4,5) d) (5,4)

iii) What are the coordinates of R? 1

a) (6,5) b) (5,6) c) ( 6,0) d) (7,4)

iv) What are the coordinates of D? 1

a) (16,0) b) (0,0) c) (0,16) d) (16,1)

v) What are the coordinate of P if D is taken as the origin? 1

a) ( 12,2) b ) (-12,6) c) (12,3) d) (6,10)

Page 6 of 27
19

Rahul is studying in X Standard. He is making a kite to fly it on a Sunday. Few


questions came to his mind while making the kite. Give answers to his
questions by looking at the figure.

i) Rahul tied the sticks at what angles to each other? 1


a) 30° b) 60° c) 90° d) 60°

ii) Which is the correct similarity criteria applicable for smaller triangles at the 1
upper part of this kite?

a) RHS b) SAS c) SSA d) AAS

iii) Sides of two similar triangles are in the ratio 4:9. Corresponding medians of 1
these triangles are in the ratio,

a) 2:3 b) 4:9 c) 81:16 d) 16:81

iv) In a triangle, if square of one side is equal to the sum of the squares of the 1
other two sides, then the angle opposite the first side is a right angle. This
theorem is called as,

a) Pythagoras theorem b) Thales theorem


c) Converse of Thales theorem d) Converse of Pythagoras theorem

v) What is the area of the kite, formed by two perpendicular sticks of length 6 cm 1
and 8 cm?

a) 48 cm2 b) 14 cm2 c) 24 cm2 d) 96 cm2

Page 7 of 27
20 Due to heavy storm an electric wire got bent as shown in the figure. It followed
a mathematical shape. Answer the following questions below.

i) Name the shape in which the wire is bent 1

a) Spiral b) ellipse c) linear d) Parabola

ii) How many zeroes are there for the polynomial (shape of the wire) 1

a) 2 b) 3 d) 1 d) 0

iii) The zeroes of the polynomial are 1

a) -1, 5 b) -1, 3 c) 3, 5 d) -4, 2

iv) What will be the expression of the polynomial? 1

a) x2+2x -3 b) x2 -2x +3 c) x2 - 2x -3 d) x2 +2x+3

v) What is the value of the polynomial if x = -1? 1

a) 6 b) -18 c)) 18 d) 0

Part –B
All questions are compulsory. In case of internal choices, attempt
anyone.

21 Find the coordinates of the point which divides the line segment joining the 2
points (4, -3) and (8,5) in the ratio 3:1 internally.

Page 8 of 27
OR

Find a relation between x and y such that the point (x,y) is equidistant from the
points (7,1) and (3,5)

22 2

AM AN
In the fig. if LM II CB and LN II CD, prove that =
MB ND

23 A quadrilateral ABCD is drawn to circumscribe a circle. Prove that AB + CD = 2


AD + BC.

24 Draw a line segment of length 7.8 cm and divide it in the ratio 5:8. Measure 2
the two parts.

25 Given 15 cot A =8, find sin A and sec A. 2

OR

Find tan P – cot R

Page 9 of 27
26 How many terms of the A. P : 9,17,25, ....... must be taken to give a sum 636? 2

Part –B
All questions are compulsory. In case of internal choices, attempt
anyone.

27 Prove that √3 is an irrational number. 3

28 Two tangents TP and TQ are drawn to a circle with centre O from an external 3
point T. Prove that PTQ = 2OPQ.

29 Meena went to a bank to withdraw Rs.2,000. She asked the cashier to give 3
her Rs.50 and Rs.100 notes only. Meena got 25 notes in all. Find how many
notes of Rs.50 and Rs.100 she received.

30 A box contains 90 discs which are numbered from 1 to 90. If one disc is drawn 3
at random from the box, find the probability that it bears
(i) a two-digit number
(ii) a perfect square number.
(iii) a number divisible by 5.

OR

One card is drawn from a well shuffled deck of 52 cards. Find the probability of
getting
(i) A king of red colour.
(ii) A spade
(iii) The queen of diamonds

31 Metallic spheres of radii 6cm, 8cm and 10cm respectively are melted to form a 3
solid sphere. Find the radius of the resulting sphere.

Page 10 of 27
32 sin A−cosA+1 1 3
Prove that =
sinA+cosA−1 secA−tanA

33 A motor boat whose speed in still water is 18 km/h, takes 1 hour more to go 24 3
km upstream than to return downstream to the same spot. Find the speed of
the stream.

OR

Find two consecutive odd positive integers, sum of whose squares is 290.

Part –B
All questions are compulsory. In case of internal choices, attempt
anyone.

34 The angles of depression of the top and bottom of a 8m tall building from the 5
top of a multi storied building are 30° and 45°, respectively. Find the height of
the multi storied building and the distance between the two buildings.

OR

A 1.2m tall girl spots a balloon moving with the wind in a horizontal line at a
height 88.2 m from the ground. The angle of elevation of the balloon from the
eyes of the girl at any instant is 60°.After sometime, the angle of elevation
reduces 30°.Find the distance travelled by the balloon during the interval.

35 The pth, qth and rth terms of an A.P. are a, b and c respectively. 5
Show that a(q – r) + b(r-p) + c(p – q) = 0

Page 11 of 27
36 A survey regarding the heights in (cm) of 51 girls of class X of a school was 5
conducted and the following data was obtained. Find the median height and
the mean using the formulae.

Height (in cm) Number of Girls


Less than 140 4
Less than 145 11
Less than 150 29
Less than 155 40
Less than 160 46
Less than 165 51

Page 12 of 27
Class- X

Mathematics-Basic (241)

Marking Scheme SQP-2020-21

Max. Marks: 80 Duration:3hrs

1
1 156 = 22 x 3 x 13

2 Quadratic polynomial is given by x2 - (a +b) x +ab 1


x2 -2x -8

3 HCF X LCM =product of two numbers ½


LCM (96,404) = 96 x 404 = 96 x 404
HCF( 96,,404 ) 4 ½
LCM = 9696

OR

Every composite number can be expressed (factorized) as a product 1


of primes, and this factorization is unique, apart from the order in
which the factors occur.

4 x – 2y =0

3x + 4y -20 =0

1 −2
≠ ½
3 4

b1
As, a1 ≠ is one condition for consistency.
a2 b2

Therefore, the pair of equations is consistent. ½

5 1 1

6 ɵ = 60°
Area of sector = ɵ Πr2
360°
60° 22 ½
A= X X (6)2 cm2
360° 7
A = 1 X 22 X36 cm2
6 7
= 18.86cm2 ½Page 13 of 27
OR

Another method-
Horse can graze in the field which is a circle of radius 28 cm. ½
So, required perimeter = 2πr= 2.π(28) cm
=2 x 22 X (28)cm
7
½
= 176 cm

7 By converse of Thale‟s theorem DE II BC


ADE = ABC = 70° ½
Given BAC = 50°
ABC + BAC +BCA =180° (Angle sum prop of triangles)
700 + 500 + BCA = 180°
½
BCA = 180° - 120° = 60°

OR

EC = AC – AE = (7- 3.5) cm = 3.5 cm


AD 2 3.5 1 ½
= and AE = =
BD 3 EC 3.5 1
AD
So, ≠ AE
BD EC ½
Hence, By converse of Thale‟s Theorem, DE is not Parallel to BC.
8 TOTAL COST
Length of the fence =
RATE
.5280
= = 220 m ½
24/
So, length of fence = Circumference of the field
∴ 220m= 2 Π r=2 X 22 x r
7
220 X 7
So, r = m =35 m ½
2 X 22

Sol: tan 30 ° = AB ½
BC
1/√3 = AB

8
AB = 8 / √3 metres ½
Height from where it is broken is 8/√3 metres

Page 14 of 27
10 Perimeter = Area 1
2πr = πr2
r = 2 units

11 3 median = mode + 2 mean 1

12 8 1

13 a1 b1 ½
≠ is the condition for the given pair of equations to have unique
a2 b2
solution.

4
≠ p
2 2

p ≠4 ½

Therefore, for all real values of p except 4, the given pair of equations
will have a unique solution.

OR

a1 2 1
Here, = =
a2 4 2

a1 3 1 c1 5
= = and =
a2 6 2 c2 7

1 1 5
= ≠
2 2 7 ½
a1 b1 c1
= ≠ is the condition for which the given system of equations
a2 b2 c2
will represent parallel lines.
½
So, the given system of linear equations will represent a pair of parallel
lines.
14 No. of red balls = 3, No.black balls =5 ½
Total number of balls = 5 + 3 =8
Probability of red balls =
3 ½
8

OR

Total no of possible outcomes = 6 ½


There are 3 Prime numbers, 2,3,5. ½
3
So, Probability of getting a prime number is =1
6 2

Page 15 of 27
15

tan 60° = h
15
√3 = h
15
h = 15√3 m ½

16 1 1

17 i) Ans : b) 1
Cloth material required = 2X S A of hemispherical dome
= 2 x 2π r2
= 2 x 2x 22 x (2.5)2 m2
7
= 78.57 m2

ii) a) Volume of a cylindrical pillar = π r2h 1

iii) b) Lateral surface area = 2x 2πrh 1


22 2
= 4 x x 1.4 x 7 m
7
= 123.2 m2

iv) d) Volume of hemisphere = 2


π r3 1
3
= 2 22 (3.5)3 m3
3 7
= 89.83 m3

v)
b)
Sum of the volumes of two hemispheres of radius 1cm each= 2 x 2 π13
3
½
Volume of sphere of radius 2cm = 4
π 23
3
2
2x π13
3
So, required ratio is 4 = 1:8 ½
π 23
3

Page 16 of 27
18 i) c) (0,0) 1

ii) a) (4,6) 1

iii) a) (6,5) 1

iv) a) (16,0) 1

v) b) (-12,6) 1

19 i) c) 90° 1

ii) b) SAS 1

iii) b) 4 : 9 1

iv) d) Converse of Pythagoras theorem 1

v) a) 48 cm2 1

20 i) d) parabola 1

ii) a) 2 1

iii) b) -1, 3 1

iv) c) x2 − 2 − 3 1

v) d) 0 1

21 Let P(x,y) be the required point. Using section formula

m1x2+m2x1 m1y2+m2y1 1
{ , } = (x, y)
m1+m2 m1+m2
3(8)+1(4) 3(5)+1(−3)
x= , y=
3+1 3+1
x=7 y= 3 1
(7,3) is the required point

Page 17 of 27
OR

Let P(x, y) be equidistant from the points A(7,1) and B(3,5) 1


Given AP =BP. So, AP2 = BP2
(x-7)2 + (y-1)2 = (x-3)2 + (y-5)2
2 2 2
X2 -14x+49 +y -2y +1 = x -6x +9+y -10y+25
1
x – y =2

22 By BPT, ½
AM = AL ............(1)
MB LC

................. ½
Also, AN = 𝐴𝐿 (2)
ND LC

By Equating (1) and (2) AM =𝐴𝑁 1


MB ND

23 To prove: AB + CD = AD + BC.

1
Proof: AS = AP ( Length of tangents from an external point to a circle
are equal)
1
BQ = BP
CQ = CR
DS = DR
AS + BQ + CQ + DS = AP + BP + CR + DR
(AS+ DS) + ( BQ + CQ) = ( AP + BP) + (CR + DR)
AD + BC = AB +CD

24 For the correct construction 2

Page 18 of 27
25 15 cot A =8, find sin A and sec A.
Cot A =8/15 1

Adj
=8/15
Opposite
By Pythagoras Theorem

AC2 =AB2 +BC2


½
AC =√(8x)2 + (15x)2
AC= 17x
½
Sin A = 15/17
Cos A =8/17

OR

By Pythagoras Theorem
QR = √(13)2 − (12)2 cm 1
QR = 5cm

Tan P =5/12
Cot R =5/12 1
Tan P -Cot R =5/12 -5/12
=0

26 9,17,25, .......
Sn = 636
a=9 ½
d = a2 -a1
= 17 – 9 = 8

Sn = n [ 2a + (n-1) d]
2
Sn = n [ 2a + (n-1) d]
2 ½

Page 19 of 27
636 = n [ 2x 9 + (n-1) 8]
2
1272 = n [ 18 + 8n -8]
1272 = n [10 +8n]
8n2 +10n -1272 =0
4n2 + 5n -636 =0

−b±√b2 −4ac
½
n=
2a
−5 ±√52 −4x 4x (−636)
n=
2�4
−5 ± 101
n =-
8
96 −106
n= n=
8 8
n=12 n = -−53
4
½
n=12 (since n cannot be negative)

27 Let √3 be a rational number.


Then √3 = p/q HCF (p,q) =1 1
Squaring both sides
(√3)2 = (p/q)2
3 = p2/ q2
3q2 = p2
3 divides p2 » 3 divides p
3 is a factor of p
Take p = 3C ½
3q2 = (3c)2
3q2 = 9C2
3 divides q2 » 3 divides q ½
3 is a factor of q
Therefore 3 is a common factor of p and q
It is a contradiction to our assumption that p/q is rational. 1
Hence √3 is an irrational number.

28

Page 20 of 27
Required to prove -: PTQ = 2OPQ 1
Sol :- Let PTQ = ɵ
Now by the theorem TP = TQ. So, TPQ is an isosceles triangle
TPQ = TQP = ½ (180° -ɵ) 1
= 90° - ½ ɵ
OPT = 90° ½
OPQ =OPT -TPQ =90° -(90° - ½ ɵ)
=½ɵ
= ½ PTQ ½

PTQ = 2OPQ

29 Let Meena has received x no. of 50 re notes and y no. of 100 re 1


notes.So,
50 x + 100 y =2000
x + y =25
multiply by 50
1
50x + 100y =2000
50 x + 50 y = 1250
- - -
50y =750
Y= 15
1
Putting value of y=15 in equation (2)
x+ 15 =25
x = 10

Meena has received 10 pieces 50 re notes and 15 pieces of 100 re


notes

30 (i) 10,11,12…90 are two digit numbers. There are 81


numbers.So,Probability of getting a two-digit number 1
= 81/90 =9/10

(ii) 1, 4, 9,16,25,36,49,64,81 are perfect squares. So, 1


Probability of getting a perfect square number.
= 9/90 =1/10

(iii) 5, 10,15….90 are divisible by 5. There are 18 outcomes.. 1


So,Probability of getting a number divisible by 5.
= 18/90 =1/5

Page 21 of 27
OR

(i) Probability of getting A king of red colour. 1

P (King of red colour) = 2/52 =1/26

(ii) Probability of getting A spade 1


P ( a spade) = 13/52 = 1/4

(iii) Probability of getting The queen of diamonds 1


P ( a the queen of diamonds) = 1/52

31 r1 = 6cm
r2 = 8cm
r3 = 10cm
1
Volume of sphere = 4/3 π r3
Volume of the resulting sphere = Sum of the volumes of the smaller
spheres.
3
4/
3π r = 4/3 π r 3 + 4/ π r 3 +4/ π r 3 3 1
1 3 2 3
3
4/
3π r = 4/3 π (r 3 + r 3 + r3 3)
1 2
r 3 = 63 + 83 + 103
r3 = 1728
r = 3√1728
r = 12 cm
1

Therefore, the radius of the resulting sphere is 12cm.

32 (sin A-cos A+1)/ (sin A+cosA-1) = 1/(sec A-tan A)

L.H.S. divide numerator and denominator by cos A


1
= (tan A-1+secA)/ (tan A+1-sec A)

= (tan A-1+secA)/(1-sec A + tan A)

We know that 1+tan2 A=sec 2A 1


Or 1=sec2 A-tan2 A = (sec A + tan A)(sec A – tan A)

=( sec A + tan A-1)/[(sec A + tan A)(sec A-tan A)-(sec A-tan A)]

=( sec A + tan A-1)/(sec A-tan A)(sec A + tan A-1)


1

Page 22 of 27
= 1/(sec A-tan A) , proved.

33 Given:-

Speed of boat =18km/hr


Distance =24km

Let x be the speed of stream. ½


Let t1 and t2 be the time for upstream and downstream.
As we know that,

speed= distance / time


⇒time= distance / speed
½
For upstream,
Speed =(18−x) km/hr
Distance =24km
Time =t1
Therefore,

24
t1 =
18−x
For downstream,
Speed =(18+x)km/hr
Distance =24km
Time =t2
Therefore,

24
t2 =
18+x
Now according to the question-

t1=t2+1
24 24
= +1
18−x 18+x
24(18+x)− 24 (18−x ) ½
⇒ =1
(18−x)(18+x)

⇒48x=(18−x)(18+x)

⇒48x=324+18x−18x− x2

⇒ x2 +48x−324=0
⇒ x2+54x−6x−324=0
⇒x(x+54)−6(x+54)=0
⇒(x+54)(x−6)=0

Page 23 of 27
½
⇒x=−54 or x=6

Since speed cannot be negative.

⇒x=−54 will be rejected

∴ x=6

Thus, the speed of stream is 6km/hr. 1

OR

Let one of the odd positive integer be x


then the other odd positive integer is x+2 1
their sum of squares = x² +(x+2)²
= x² + x² + 4x +4
= 2x² + 4x + 4
Given that their sum of squares = 290
⇒ 2x² +4x + 4 = 290
⇒ 2x² +4x = 290-4 = 286
⇒ 2x² + 4x -286 = 0 1
⇒ 2(x² + 2x - 143) = 0
⇒ x² + 2x - 143 = 0
⇒ x² + 13x - 11x -143 = 0
⇒ x(x+13) - 11(x+13) = 0
⇒ (x -11)(x+13) = 0
⇒ (x-11) = 0 , (x+13) = 0
Therefore , x = 11 or -13
According to question, x is a positive odd integer.
Hence, We take positive value of x 1
So , x = 11 and (x+2) = 11 + 2 = 13
Therefore , the odd positive integers are 11 and 13 .

Page 24 of 27
34

Let AB and CD be the multi-storeyed building and the building


respectively.

Let the height of the multi-storeyed building= h m and

the distance between the two buildings = x m.

AE = CD = 8 m [Given]

BE = AB – AE = (h – 8) m

and

AC = DE = x m [Given]

Also,

∠ FBD = ∠ BDE = 30° ( Alternate angles)

∠ FBC = ∠ BCA = 45° (Alternate angles) ½

Now,

In Δ ACB,

In Δ BDE,

Page 25 of 27
1

From (i) and (ii), we get,

h =√3h -8√3

√3h – h =8√3

h (√3 -1) =8√3

8√3
h=
√3−1

8√3
h= x√3+1 1
√3−1 √3+1

h-= 4√3 (√3 +1)

h = 12 +4√3 m

Distance between the two building


½

OR

E C

D
A B

From the figure, the angle of elevation for the first position of the
balloon EAD = 60° and for second position BAC = 30°.The
vertical distance
1
ED = CB = 88.2-1.2 =87m.

Page 26 of 27
Let AD = x m and AB = y m.

Then in right Δ ADE, tan60° = DE


AD

√3 = 87
1
X

..............
X = 87 (i)
√3

In right ΔABC, tan 30° = BC


AB

1 87
=
√3 Y

Y = 87√3 ............ (ii)


Subtracting(i) and (ii) 1
y-x =87√3 - 87
√3

87 - 2.√3
y-x = 1
√3 .√3

y-x = 58√3 m

Hence, the distance travelled by the balloon is equal to BD

y-x =58√3 m.

35 Let A be the first term and D the common difference of A.P.

Tp=a=A+(p−1)D=(A−D)+pD (1)
½
Tq=b=A+(q−1)D=(A−D)+qD ..(2) ½

Tr=c=A+(r−1)D=(A−D)+rD ..(3) ½

Here we have got two unknowns A and D which are to be eliminated.

We multiply (1),(2) and (3) by q−r,r−p and p−q respectively and add:

a (q-r) = (A – D )(q-r) + D p(q-r) ½


b(r-p) = (A-D) (r-p) + Dq (r-p) ½
c(p-q) = (A-D) (p-q) + Dr (p-q) ½

a(q−r)+b(r−p)+c(p−q)
1
=(A−D)[q−r+r−p+p−q]+D[p(q−r)+q(r−p)+r(p−q)]
= (A – D ) ( 0 ) + D [ pq-pr + qr – pq + rp – rq ) 1
=0

Page 27 of 27
36 Height (in cm) f C.F.
below 140 4 4
140-145 7 11 1
145-150 18 29
150-155 11 40
155-160 6 46
160-165 5 51
N=51⇒

N/2=51/2=25.5

As 29 is just greater than 25.5, t herefore median class is 145-150.


N
( −Cf)
2
Median= l + Xh
f

Here, l= lower limit of median class =145

C=C.F. of the class preceding the median class =11 ½


h= higher limit - lower limit =150 −145=5

f= frequency of median class =1 8

∴ median=
( 25.5−11)
= 145 + X5
18 ½
=149.03
Mean by direct method
1
f xi fxi
Height (in cm)
below 140 4 137.5 550
140-145 7 142.5 997.5
145-150 18 147.5 2655
150-155 11 152.5 1677.5
155-160 6 157.5 945
5 162.5 812. 1
160-165 ∑ fx
Mean =
N

=7637 .5/51
= 149.75
1
@KVSHQ
@KVS_HQ

DESIGNED & PRINTED BY :


CHOUDHARY PRINTING PRESS, Near Mohanpur Devisthan, Punaichak, Patna-800 023
Mob. : 09430906087, 07903299242, T/F: 0612-2546751
E-mail - [email protected], Website : www.choudharyprintingpress.com

You might also like